Чемпионат: Кубок Москвы по ЧГК. Гран-При сезона 2011/12 гг. Дата: 00-000-2011 Тур: 1 тур. Команда Ивана Семушина Дата: 18-Dec-2011 Редактор: Иван Семушин, Александра Брутер, Максим Руссо Инфо: Редакторы пакета благодарят за тестирование вопросов Константина Науменко, Владимира Островского, Александра Толесникова, Сергея Коновалова, Светлану Орлову, Александра Салиту, Юрия Выменца и Наталью Кудряшову, а также Алексея Гилёва. Вопрос 1: Владимир Ткаченко в песне "Бабл-гам" рифмует слова, которые рифмуются и в стихотворении 1917 года. Напишите эти слова. Ответ: Жуй, буржуй. Источник: 1. http://www.txtmusic.ru/view.php?id=32494 2. http://ru.wikisource.org/wiki/Ешь_ананасы,_рябчиков_жуй_(Маяковский) Автор: Иван Семушин Вопрос 2: В англо-русском словаре можно увидеть, что слово, обозначающее ПЕРВЫЕ, переводится в том числе как "ветки", а слово, обозначающее ВТОРЫЕ, - как "шишки". Как называют ПЕРВЫЕ и ВТОРЫЕ у нас? Ответ: Палочки и колбочки. Зачет: Фоторецепторы. Источник: 1. http://en.wikipedia.org/wiki/Rod_cell 2. http://en.wikipedia.org/wiki/Cone_cell 3. http://lingvo.yandex.ru/rod/с%20английского/ 4. http://lingvo.yandex.ru/cone/с%20английского/ Автор: Александра Брутер Вопрос 3: Антуан де Сент-Экзюпери признавался, что после того, как он выучился на летчика, ему очень пригодилась ОНА. Тем самым он вольно или невольно подтвердил правоту одной из сторон в знаменитом споре. Назовите фамилию той, которая отстаивала противоположную точку зрения. Ответ: Простакова. Комментарий: Экзюпери говорил о географии. Источник: 1. http://malenkiyprinc.narod.ru/1.html 2. http://www.flibusta.net/b/169510/read Автор: Илья Иткин Вопрос 4: Говоря о падении культурного уровня, Иван Зорин пишет, что современный обыватель в энциклопедии может разобрать только пьесу. Какое слово мы заменили в предыдущем предложении? Ответ: Клоп. Комментарий: В энциклопедии слышится только "клоп". Источник: http://zorinivan.livejournal.com/4490.html Автор: Иван Семушин Вопрос 5: Слово "ajtajtó" по-венгерски означает "дверь". Как венгерский ювелир по фамилии Айтоши, переехавший в Германию, назвал своего второго сына? Ответ: Альбрехт. Комментарий: Отец художника буквально перевел свою фамилию на немецкий как Türer; впоследствии она стала записываться как Dürer. Источник: http://ru.wikipedia.org/wiki/Дюрер,_Альбрехт Автор: Александра Брутер Вопрос 6: В некоторых греческих музеях демонстрируются видеоролики, в которых современные актеры разыгрывают сцены из древней жизни. Ролики, как правило, хорошо подготовлены консультантами-историками, однако при просмотре иногда можно заметить один анахронизм. Ответьте, использовав аббревиатуру, что появилось в начале XX века благодаря Кальметту и Герену. Ответ: Прививка БЦЖ. Зачет: Прививка BCG (зачет по аббревиатуре). Комментарий: У актеров на руках следы противотуберкулезной прививки, известной как БЦЖ (Bacillus Calmette-Guérin). Источник: 1. Личные наблюдения автора вопроса. 2. http://ru.wikipedia.org/wiki/БЦЖ Автор: Александра Брутер Вопрос 7: <раздатка> Задолго до Никоновой порухи было на Руси лихолетье... - начал рассказ свой старец Иосиф. Перед вами цитата из романа Мельникова-Печерского "В лесах", в которой мы заменили одну букву. Восстановите первоначальный вид измененного нами слова. Ответ: Ляхолетье. Комментарий: Так иронически называли на Руси Смутное время. Источник: http://www.e-reading.org.ua/chapter.php/148232/76/Mel'nikov-Pecherskiii_-_V_lesah.html Автор: Илья Иткин Вопрос 8: Прослушайте две строчки из стихотворения Тарковского: Был язык мой правдив, как [пропуск], А слова у меня под ногами валялись. При помощи того, что мы пропустили, был открыт второй. Заполните пропуск. Ответ: Спектральный анализ. Комментарий: С помощью него был открыт гелий - второй элемент периодической таблицы. Источник: 1. http://www.litera.ru/stixiya/authors/tarkovskij/ya-uchilsya-trave.html 2. http://ru.wikipedia.org/wiki/Спектральный_анализ Автор: Александра Брутер Вопрос 9: Сетуя на низкий уровень образования современных священников, Георгий Нипман говорит, что они не только не имеют представления о том, кто написал "Руководство к христианской жизни", но и не могут ответить даже на более простой вопрос. Этот вопрос из трех слов фигурирует и в известном стихотворении. Что это за вопрос? Ответ: "Кто такой Савонарола?". Комментарий: Речь идет о стихотворении Бродского "Представление". Упомянутое в вопросе произведение, собственно, принадлежит Савонароле. Источник: 1. http://magazines.russ.ru/volga/2009/3/ni8.html 2. http://lib.rus.ec/b/66421/read Автор: Александра Брутер Вопрос 10: Цитата из Иэна Макьюэна: "Вставало солнце, облаков почти не было. Идеальный день для...". Закончите цитату словом с тремя буквами "ф". Ответ: Люфтваффе. Комментарий: Действие этой части романа происходит во время Второй мировой войны, события показаны со стороны воюющего во Франции британского солдата. Источник: http://www.flibusta.net/b/98242/read Автор: Иван Семушин Вопрос 11: <раздатка> "_____ ___ and the Little Boys" Комикс "Хранители" пронизан темой ядерной опасности. В одном из кадров можно даже обнаружить афишу представления в клубе для гомосексуалистов с надписью, которую мы вам раздали. Какие два слова мы в ней пропустили? Ответ: Enola Gay. Комментарий: Название самолета, сбросившего бомбу на Хиросиму. "Little Boy" - это та самая бомба, "Малыш". Источник: Комикс "Watchmen", гл. 2. Автор: Иван Семушин Вопрос 12: Как известно, барон Мюнхгаузен часто искажает истину. Так, в книге Кржижановского он упоминает АЛЬФЫ фонтанных струй. Что мы заменили на АЛЬФУ? Ответ: Гипербола. Комментарий: Вообще-то струя теоретически должна иметь форму параболы. А преувеличивать барон тоже любит. Источник: http://www.flibusta.net/b/169710/read Автор: Иван Семушин Вопрос 13: <раздатка> Вот старый нелепый альбомчик предреволюционной барышни для вписывания приветов с вензелями Е.В. на черной гофрированной обложке, вот строки полинялых фиолетовых чернил - посвящение на память милой, славной Кате, поэтичному бесенку (и Кате, и бесенку, разумеется, [два слова пропущено]). Перед вами цитата из книги Николая Климонтовича. Заполните пропуск. Ответ: "... через ять". Источник: Николай Климонтович. Фотографирование и прочие игры. - М., 1990. Автор: Илья Иткин Вопрос 14: (pic: 20110719.jpg) Перед вами изображение характерного декоративного элемента из дома-музея выдающегося представителя бельгийского ар-нуво Виктора Орта. Есть версия, что этот архитектурный стиль нашел отражение в НИХ. Назовите ИХ двумя словами. Ответ: Усы Пуаро. Комментарий: (pic: 20110720.jpg) Источник: http://la-gatta-ciara.livejournal.com/72147.html Автор: Александра Брутер Вопрос 15: В одном романе житель древнего Китая попадает в современный мир. Поначалу его пугают произошедшие за тысячу лет изменения. Но через некоторое время в доме приютившего его человека он видит предмет, связывающий его с прошлым, и настраивается на философский лад. Ответьте двумя словами, что это за предмет. Ответ: Книга перемен. Зачет: И Цзин, Канон перемен. Источник: http://www.flibusta.net/b/61804/read Автор: Иван Семушин Тур: 2 тур. Команда Ильи Бера Дата: 18-Dec-2011 Инфо: Команда благодарит за помощь в тестировании вопросов Вадима Данько, Юрия Выменца, Александра Коробейникова, Михаила Иванова. Вопрос 1: Многие фантасты, а за ними ученые говорили, что космос может стать для человечества новым домом. Похоже, серьезнее других к этой идее отнеслись французы. Ответьте абсолютно точно, что они сделали 18 октября 1963 года. Ответ: Запустили в космос кошку. Зачет: По упоминанию космоса (орбиты) и кошки. Комментарий: У многих народов есть традиция, по которой перед заселением в новый дом туда запускают кошку на удачу. Кстати, кошка Фелисетта вернулась на землю живой и невредимой. Источник: 1. http://ru.wikipedia.org/wiki/Животные_в_космосе 2. Измышления автора вопроса. Автор: Илья Бер Вопрос 2: Барбара Такман, описывая настроения лета 1914 года, замечает, что всего за несколько дней немецкие рабочие повернулись от ПЛАТОНА к ПЛУТОНУ. Какие два слова мы заменили на "ПЛАТОН" и "ПЛУТОН"? Ответ: Маркс, Марс. Источник: Tuchman B. The guns of August. London, 1962. P. 74. Автор: Сергей Вакуленко Вопрос 3: Папа Римский Мартин I, осужденный на ЭТО, жаловался друзьям на холод, нехватку хлеба и грубость местных жителей. Один генерал, вероятно, с ним бы не согласился. Назовите ЭТО тремя словами. Ответ: Ссылка в Крым. Комментарий: Генерал из "Сказки про Федота-стрельца" был согласен на ссылку, "но желательно в июле и желательно в Крыму". Источник: 1. http://www.vostlit.info/Texts/Dokumenty/Italy/VII/640-660/Martin_I/brief_16.phtml?id=5972 2. http://archeologia.narod.ru/krim/krimkalami.htm 3. http://lib.ru/ANEKDOTY/fedot.txt Автор: Станислав Мереминский Вопрос 4: Итальянское слово "casa" [кАза] означает "дом, жилище, домашнее хозяйство". Миланское издательство "La casa di Madonna" [ла кАза ди мадОнна], основанное в 1975 году, специализируется на выпуске книг по истории и культуре России. В названии издательства мы немного изменили одно слово. Напишите его в первоначальном виде. Ответ: Matriona. Зачет: Matryona, Matrena, Matrona, Матрёна. Комментарий: По-русски: "Матрёнин двор". Издательство основано вскоре после высылки Солженицына из СССР. Источник: http://www.russiacristiana.org/russiacristianarced.htm Автор: Станислав Мереминский Вопрос 5: По мнению журналиста Джереми Кларксона, запуск первого советского спутника означал, что советские ПЛАНКИ лучше, чем американские ПЛАНКИ. Какие два слова мы заменили на "ПЛАНКИ"? Ответ: Немецкие ученые. Зачет: Немецкие физики. Источник: http://f.inosmi.ru/showthread.php?t=24609&langid=1 Автор: Сергей Вакуленко, Станислав Мереминский Вопрос 6: Герой романа Патрика О'Брайена - британский офицер эпохи наполеоновских войн - беседует со своей возлюбленной. Во время разговора в комнату входит служанка. Вскоре после этого герой с удивлением сознаёт, что его возлюбленная относится к нему гораздо лучше, чем он предполагал. Напишите короткое слово, которое мешало ему понять это раньше. Ответ: You. Комментарий: Чтобы разговор не поняла служанка, герои переходят на французский, в котором различаются местоимения "ты" и "вы". Английское местоимение "you" не дает возможности понять степень интимности обращения. Источник: http://www.e-reading.org.ua/bookreader.php/135169/The_Fortune_of_War.html Автор: Станислав Мереминский Вопрос 7: На советской карикатуре 1952 года ОН изображен в виде яйца, из которого выползают ядовитые змеи. Автор ЕГО проекта - Томас Уолтер. Назовите ЕГО двумя словами, начинающимися на одну и ту же букву. Ответ: Купол Капитолия. Зачет: Купол конгресса. Источник: 1. "Крокодил", 1952, N 1. 2. http://en.wikipedia.org/wiki/United_States_Capitol_dome Автор: Станислав Мереминский Вопрос 8: Единственным судном ВМФ Соединенных Штатов Америки, совершившим кругосветное плавание, стал во второй половине XIX века клипер "Шенандоа" под командованием капитана Джеймса Уоддела. Какое слово мы заменили в предыдущем предложении? Ответ: Конфедеративных. Зачет: Конфедерации. Комментарий: Судов ВМФ США, обогнувших Землю, конечно, очень много. Источник: 1. http://en.wikipedia.org/wiki/List_of_circumnavigations 2. http://en.wikipedia.org/wiki/CSS_Shenandoah Автор: Станислав Мереминский Вопрос 9: Температура среды обитания Champsocephalus gunnari [хампсоцефАлус гУннари] редко превышает 1-2 градуса Цельсия, однако ее русское название связано не с этим, а с очень малым количеством эритроцитов. Напишите это русское название из двух слов. Ответ: Ледяная рыба. Зачет: Щуковидная белокровка. Комментарий: Кровь ледяной рыбы практически прозрачна, отсюда и ее название, хотя и живет она в ледяной воде. Источник: http://www.zoopicture.ru/ledyanaya-ryba/ Автор: Станислав Мереминский Вопрос 10: [Ведущему: интонацией кавычки и капитализацию не обозначать!] По одной из версий, ЕГО русское название происходит от чувашского слова со значением "убегающий". Назовите того из НИХ, который однажды напился и потому проиграл в "Беге". Ответ: Янычар. Комментарий: По одной из версий, слово "таракан" происходит из чувашского "tar-aqan" (убегающий). В пьесе Булгакова "Бег" таракана Янычара, который был фаворитом тараканьих бегов, опоили пивом, и он позорно продул. Источник: 1. http://ru.wikipedia.org/wiki/Таракановые 2. http://lib.ru/BULGAKOW/run.txt Автор: Илья Бер Вопрос 11: По мнению историка Романенко, Николай МАйков в своей деятельности основывался на традициях и принципах египетского монашества. Под каким именем стал известен Николай МАйков? Ответ: Нил [Сорский]. Источник: http://www.sedmitza.ru/text/443550.html Автор: Станислав Мереминский Вопрос 12: На известный риторический вопрос могла бы утвердительно ответить, например, Грейс Фиш, муж которой - Фред Фиш - благодаря удачному браку в 1891 году возглавил крупную компанию по производству повозок и фургонов. Напишите девичью фамилию Грейс Фиш. Ответ: Студебеккер. Комментарий: Грейс была дочерью Джона Студебеккера, одного из основателей одноименной компании (кстати, начавшей производство автомобилей как раз по инициативе Фреда Фиша). Вопрос Остапа Бендера: "Папа ваш Студебеккер?". Источник: 1. http://lib.ru/ILFPETROV/telenok.txt 2. http://www.findagrave.com/cgi-bin/fg.cgi?page=gr&GRid=65313008 Автор: Станислав Мереминский Вопрос 13: <раздатка> 1. Рассыплет скрипка свои коленца, Рассвет укроет сырой периной, Неотвратимый, как инфлюэнца. С напрасной горечью аспирина. 2. Ну что ж - обещанного ждем. А умываемся покуда Водой - что делать? - ледяной, почти колодезной, когда б Не [ПРОПУСК] на губах, не сиплое гуденье в трубах, И за стеной могучий ор кухонных баб... По мнению ЖЖ-юзера ulex, в стихотворении, откуда взято первое четверостишие, есть сильный ОН. ОН на одну букву короче, чем пропуск из второго четверостишия. Назовите ЕГО двумя словами. Ответ: Привкус Лорки. Комментарий: Во втором отрывке пропущены слова "привкус хлорки". Источник: 1. http://hannahlit.livejournal.com/283666.html?thread=5287442#t5287442 2. http://www.votintseva.ru/view_text.php?id=25 Автор: Илья Бер Вопрос 14: Исходя из формальной логики, у коспАка этот показатель должен был бы составлять 1,75, а у кохЕрта - 1,25. У кого он составляет 2? Ответ: Бактриан. Зачет: Двугорбый верблюд. Комментарий: КоспАк - помесь нара (гибрида дромадера и бактриана) и бактриана, кохЕрт - нара и дромадера. Показатель - число горбов. Источник: http://ru.wikipedia.org/wiki/Нар_(верблюд) Автор: Станислав Мереминский Вопрос 15: [Ведущему: многоточие в цитате не указывать.] "Древнеегипетская "Книга мертвых" ... содержала в себе обещание воскрешения тому, кто будет похоронен с ее копией". Напишите название статьи Википедии, в начале которой приводится эта информация. Ответ: Письма счастья. Зачет: Письмо счастья, цепное письмо, цепные письма. Незачет: "нигерийские письма" - это другое явление. Комментарий: Архетип, на котором построены подобные письма, оказывается, очень древний. Обычно истоки, история какого-либо явления или события приводятся в Википедии в начале статей. Источник: http://ru.wikipedia.org/wiki/Письма_счастья Автор: Илья Бер Тур: 3 тур. Команда Максима Поташева Дата: 18-Dec-2011 Вопрос 1: Писатель Андрей Белянин считает, что евреи обладают выдающейся пронырливостью и способностью пролезть в любую щель. Назовите человека, которого он приводит в качестве примера. Ответ: Гарри Гудини. Источник: А. Белянин. Опергруппа в деревне. Автор: Максим Поташев Вопрос 2: (pic: 20110721.jpg) Перед вами человек, которым восхищался потомок великого композитора. Назовите фамилию того, кто изображен на фотографии. Ответ: Ливингстон. Источник: http://www.skyborn.ru/livingstone.html Автор: Максим Поташев Вопрос 3: Персонаж одного современного автора убил 17 женщин и ненавидит Льва Толстого. Кто он по профессии? Ответ: Машинист. Источник: Слава Сэ. Жираф. Автор: Максим Поташев Вопрос 4: В известной истории АЛЬФЫ встретились друг с другом на лугу и были соединены прутьями. АЛЬФУ можно приравнять к полутора БЕТАМ. Назовите БЕТУ. Ответ: Пешка. Источник: 1. http://www.teremok.in/Pisateli/Zarub_Pisateli/raspe/mjunhauzen.htm 2. http://ru.wikipedia.org/wiki/Шахматные_фигуры Автор: Вадим Карлинский и Максим Поташев Вопрос 5: В время съемок одного из эпизодов актер Николай Яковченко несколько часов подряд выплевывал в тарелку кусочки теста. О каком фильме идет речь? Ответ: "Вечера на хуторе близ Диканьки". Зачет: "Ночь перед Рождеством". Комментарий: В фильме пленку, естественно, пустили задом наперед, и галушки стали прыгать пузатому Пацюку в рот, как оно и было в оригинале. Источник: Андрей Макаревич. Наше Вкусное Кино. - М.: КоЛибри, 2011. - С. 25. Автор: Антон Чернин Вопрос 6: У сперматозоида есть ОНА и ОН. А чья ОНА - на самом деле ОН? Ответ: Рака. Комментарий: У сперматозоида есть шейка и хвост. Раковая шейка - это на самом деле хвост. Источник: http://www.eurolab.ua/anatomy/42/ Автор: Максим Поташев Вопрос 7: (pic: 20110722.jpg) По мнению Герцена, всё, что в Италии красиво, в то же время и бессмысленно. Повсюду чувствуется рука гениального ребенка - и нигде - ОНА. На розданной иллюстрации вы можете увидеть ЕЕ. Назовите произведение, из которого взята эта иллюстрация. Ответ: "Палец инженера". Комментарий: А увидеть можно руку инженера. Источник: 1. http://commons.wikimedia.org/wiki/File:Engr-07.jpg 2. http://dic.academic.ru/dic.nsf/latin_proverbs/822/ Автор: Максим Поташев Вопрос 8: По мотивам произведений француза лет сорок назад была написана пьеса, название которой совпадает с названием произведения итальянца. В каком театре она до сих пор идет? Ответ: Театр кукол им. С. Образцова. Комментарий: Пьеса "Божественная комедия" написана Штоком по мотивам рисунков Эффеля. Источник: 1. http://ru.wikipedia.org/wiki/Жан_Эффель 2. http://ru.wikipedia.org/wiki/Шток,_Исидор_Владимирович Автор: Максим Поташев Вопрос 9: А в груди попеременно Был то [первый пропуск], то [второй пропуск]; То лукаво, то надменно Ночь прищуривала глаз. Вы прослушали четверостишие Эдмунда Мечиславовича Шклярского. Напишите два пропущенных нами слова. Ответ: Пепел, алмаз. Комментарий: Он поляк, и с польской культурой хорошо знаком. Источник: http://www.piknik.info/lyrics/index/album/19 Автор: Антон Чернин Вопрос 10: Дуплет. 1. В антифашистском фельетоне Валентина Катаева рассказывается о том, как нацисты подвергли чистке животный мир. Практически все животные были признаны евреями, подлежащими немедленному искоренению. Слон - за длинный нос, лев - за курчавость, кобра - за то, что носит пенсне. А вот киту еврейское происхождение не инкриминировали, но и его признали виновным. В чем? 2. В антифашистском фельетоне Валентина Катаева рассказывается о том, как нацисты подвергли чистке животный мир. Практически все животные были признаны евреями, подлежащими немедленному искоренению. Слон - за длинный нос, лев - за курчавость, кобра - за то, что носит пенсне. Не пострадал только ОН, и не потому, что ОН коричневый. Кто ОН? Ответ: 1. В укрывательстве еврея. 2. Клоп - в нем течет чистая арийская кровь. Комментарий: - Кит. - С корнем! - Еврей, что ли? - Кит не еврей, но три дня скрывал в своем чреве еврея без прописки. С корнем! ... - А это что? - Это клоп-с... Прикажете вырвать с корнем? Профессор побагровел. - Вы, кажется, забываете, - воскликнул он патетическим голосом, - что в жилах этого благородного животного течет чисто арийская кровь! Оставить. Посадить в лучшей клетке. Поить. Кормить. Одевать. Обувать. На казенный счет. Мы должны поощрять всех, в ком течет чисто арийская кровь. Источник: http://www.gramotey.com/?open_file=1269072661 Автор: Михаил Левандовский Вопрос 11: Лидер норвежской языческой блэк-металлической группы "Burzum" Варг Викернес провел в тюрьме 16 лет за убийство коллеги и поджоги нескольких церквей. Фамилия Викернес у него своя, а вот имя Варг - это псевдоним. Свое настоящее имя, довольно обычное для Норвегии, он отверг. Как же его зовут на самом деле? Ответ: Кристиан. Комментарий: Стремно как-то язычнику и поджигателю церквей жить с таким именем. А в Норвегии оно и впрямь встречается часто, там даже столица когда-то называлась Христианией. Источник: http://ru.wikipedia.org/wiki/Викернес,_Варг Автор: Антон Чернин Вопрос 12: По собственному признанию, он не хотел никого мистифицировать, но опасался, что товароведы в магазинах не смогут выговорить его фамилию. Напишите эту фамилию. Ответ: Чхартишвили. Источник: http://www.knigostock.com/viewtopic.php?f=6&t=14 Автор: Максим Поташев Вопрос 13: В 2011 году звание "Джентльмен года" российской Премьер-лиги впервые получил легионер. Во время церемонии вручения приза ведущий отметил, что этот футболист как будто сошел со страниц романа, написанного в 1975 году. Назовите этот роман. Ответ: "Град обреченный". Комментарий: Речь идет об украинском легионере Андрее Воронине. Андрей Воронин - герой "Града обреченного". Вел церемонию не безграмотный футбольный комментатор, а Владимир Гомельский. Источник: "Советский спорт" от 24.11.2011 г. - С. 2. Автор: Максим Поташев Вопрос 14: (pic: 20110723.jpg) Аппарат, который все вы видели, это делать не мог, а орган, который все вы видите, по мнению некоторых ученых, мог. Что именно делать? Ответ: Преодолевать звуковой барьер. Комментарий: Самолет Ливингстона явно не мог, а хвост апатозавра, похоже, мог. Источник: http://en.wikipedia.org/wiki/Whipcracking Автор: Максим Поташев Вопрос 15: Страбон, споря с другими античными авторами, уверял, что носорожья кожа походит на слоновью, а вовсе не на какое-то дерьмо. Какое слово мы заменили в его утверждении? Ответ: Самшит. Источник: http://orient.rsl.ru/assets/files/food/302-karkadan.pdf Автор: Илья Немец Тур: 4 тур. Команда Бориса Гуревича Дата: 18-Dec-2011 Редактор: Борис Гуревич Инфо: Редактор благодарит Максима Богатова, Антона Бочкарёва и Олега Христенко за помощь в подготовке пакета, а также Николая Сергеева, Егора Господчикова и Марию Лазареву за тестирование отдельных вопросов. Вопрос 1: Один писатель не так давно выразил надежду, что его следующая книга достигнет той планки, которую он задал предыдущей. О каком писателе идет речь? Ответ: [Павел] Санаев. Комментарий: Павел Санаев - автор повести "Похороните меня за плинтусом". В интервью он выразил надежду, что его вторая книга не окажется "ниже "Плинтуса"". Источник: http://www.fontanka.ru/2009/11/23/002/ Автор: Максим Богатов Вопрос 2: В одном из залов немецкого Эбербаха сохранились предметы, по преданию, натолкнувшие на идею одного изобретения. После внедрения этого изобретения жители Эбербаха лишились доходов от одного из своих традиционных, наряду с виноделием, занятий. По ходу действия фильма, снимавшегося в Эбербахе, его персонажи также лишаются этой работы, правда, при более трагических обстоятельствах. О какой работе идет речь? Ответ: Переписывание книг. Зачет: Переписка книг и т.п. по смыслу. Комментарий: Гигантские прессы в монастыре Эбербах натолкнули Гутенберга на идею книгопечатания, а в 1986 году в бывшем монастыре снималась картина "Имя розы", в которой монахи монастыря тоже лишились этой работы. Источник: http://sean-connery.narod.ru/rose6 Автор: Николай Сергеев Вопрос 3: По мнению ЖЖ-пользователя shiloves1, популярность ЕГО образа среди определенных лиц была связана с подходящей формой подушки. Назовите ЕГО. Ответ: Наполеон. Зачет: Наполеон Бонапарт. Комментарий: Из больничной подушки очень хорошо и быстро получается треуголка. Источник: http://borisakunin.livejournal.com/39436.html?thread=15949324#t15949324 Автор: Борис Гуревич Вопрос 4: В начале 2011 года власти Камчатки попытались отменить детский спектакль, усмотрев в одной из сцен критику недавнего решения федеральных властей. По какой сказке был поставлен этот спектакль? Ответ: "Золушка". Комментарий: "Это была сцена, в которой король переводит стрелки часов на 1 час назад, чтобы Золушка дольше осталась на балу. Намек, можно сказать, на самого "короля" Дмитрия Анатольевича. Ведь именно президент Медведев был инициатором перевода часов на Камчатке на час назад в рамках своей смелой реформаторской идеи потуже затянуть российские часовые пояса. А на Камчатке народ этому очень не обрадовался, даже проводил митинги". Источник: http://www.gazeta.ru/column/novoprudsky/3490278.shtml Автор: Борис Гуревич Вопрос 5: Однажды литератору посоветовали написать "Фауста" и, в ответ на недоуменный вопрос: "Перевести?", подчеркнули: "Нет, написать своего!". По одной из версий, после этого разговора сложилось окончательное понимание того, каким должен быть герой будущего произведения. Что это за произведение? Ответ: "Доктор Живаго". Комментарий: Ахматова советовала написать Пастернаку нового "Фауста" и предположила, что именно поэтому Живаго стал доктором. Источник: http://dozor.narod.ru/writers/berestov/faustus.html Автор: Максим Богатов Вопрос 6: (pic: 20110724.jpg) Перед вами некоторые варианты масок, созданных специально для "НЕЕ". "Сакра Корона Унита" - это тоже ОНА. Назовите ЕЕ одним словом. Ответ: Мафия. Комментарий: Глаза закрыты - отличные маски для "мафии". "Сакра Корона Унита" - название апулийской мафии (по аналогии с Козой Нострой, Каморрой и т.п.). Источник: 1. http://www.karnavalmasok.ru/view_news.php?id=27 2. http://nightlondon.mybb.ru/viewtopic.php?id=50 Автор: Николай Сергеев Вопрос 7: Восстановите два пропущенных слова в стихотворении Бориса Смоленского: Учебник в угол - и на пароход В июнь, в свободу, в ветер, в поцелуи. И только берега, как [пропуск], Стянули неба синюю кривую. Ответ: "... пара хорд...". Источник: http://www.litera.ru/stixiya/authors/smolenskij/uchebnik-v-ugol.html Автор: Борис Гуревич Вопрос 8: Семен Липкин написал биографическую книгу о своем друге. В название книги входит название самого известного романа друга. Назовите этого друга. Ответ: [Василий] Гроссман. Комментарий: "Жизнь и судьба" - роман Василия Гроссмана. А книга Липкина - биографическая, о жизни и судьбе писателя. Источник: http://www.imwerden.info/belousenko/books/Lipkin/lipkin_berzer_grossman.htm Автор: Борис Гуревич Вопрос 9: Дмитрий Быков, предрекая персонажу известного произведения судьбу остальных, сравнивает получающуюся картину с распластанной на снегу тенью птицы. О каком персонаже идет речь? Ответ: [Буква] "И". Комментарий: Лужа замерзает, арка скалится, Клонятся фонарные столбы, Тень от птицы по снегу пластается, Словно "И", упавшее с трубы. Произведение - "А и Б сидели на трубе". Источник: http://www.netslova.ru/bykov/stihi.html Автор: Борис Гуревич Вопрос 10: В Венеции купца, захватившего остров и поднявшего там флаг республики, награждали особым титулом. Но, как правило, это были пустынные острова, не имевшие никакого значения, поэтому к подобным "захватам" венецианцы относились с большой иронией. Так объясняют происхождение одного имени. Какого? Ответ: Панталоне. Комментарий: Титул звучит как Pianta-Leone (Водрузитель Льва), так как лев - символ св. Марка - изображен на венецианском флаге. Панталоне - одна из основных масок северного варианта комедии дель арте - как раз изображает старого купца. Источник: http://ru.wikipedia.org/wiki/Панталоне Автор: Евгений Дёмин Вопрос 11: <раздатка> Мне приходилось слышать, что этот поединок будет спектаклем. Ничего подобного! Гердт то и дело пытался испортить виновнику торжества настроение. Перед вами отрывок из сообщения о состоявшемся в 1971 году событии, автор которого написал одно из слов с ошибкой. Назовите виновника торжества. Ответ: [Лев] Яшин. Комментарий: Речь шла не о Зиновии Гердте, а о Герде Мюллере. Это был прощальный матч Льва Яшина. Источник: http://fakty.ua/133996-sorok-let-nazad-lev-yashin-provel-svoj-poslednij-match Автор: Борис Гуревич Вопрос 12: В результате вполне серьезного исследования ученые пришли к выводу, что шансы гигантского кальмара невелики. А кто фигурирует вместо кальмара в известном вопросе? Ответ: Слон. Комментарий: Единственным хищником, угрожающим взрослым особям гигантского кальмара, является кашалот. Ранее бытовало мнение, что между кальмарами и кашалотами разыгрываются страшные битвы, исход которых может быть успешным как для моллюска, так и для кита. На самом же деле разница в их силе и весе столь велика, что "битва" почти всегда заканчивается в пользу кашалота. Единственная серьезная опасность для кашалота - то, что кальмар может заткнуть щупальцем его дыхало... А вот отгадайте сами: если слон и вдруг на кита налезет, кто кого сборет? - Не знаю, - постыдно признался директор. - Никто не знает, - утешился Ося, - ни папа, ни солдат, никто... Источник: 1. http://ru.wikipedia.org/wiki/Гигантский_кальмар 2. http://lib.ru/PROZA/KASSIL/kassil1.txt Автор: Антон Бочкарёв Вопрос 13: Внимание, в вопросе есть замены. Британскому актеру Дэвиду Суше при подготовке к роли Эркюля Пуаро пришлось ДЕЛАТЬ ЭТО. Персонаж известного произведения профессионально помогал ДЕЛАТЬ ОБРАТНОЕ. Назовите этого персонажа. Ответ: Хиггинс. Зачет: Профессор Хиггинс. Комментарий: Суше пришлось нанять преподавателя и начать старательно портить английское произношение, чтобы добиться элегантного легкого акцента. Профессор Хиггинс из пьесы Шоу "Пигмалион" это самое произношение исправлял. Источник: 1. http://ru.wikipedia.org/wiki/Пуаро_Агаты_Кристи 2. Б. Шоу. Избранные пьесы. - М.: Просвещение, 1986. Автор: Николай Сергеев Вопрос 14: (pic: 20110725.jpg) Назовите фамилию любого из людей, изображенных на розданных вам фотографиях. Ответ: Толстой. Зачет: Жемчужников. Комментарий: Подписи неправильные. "Если на клетке слона написано "буйвол" - не верь глазам своим", как говорил К. Прутков. Источник: Фотографии братьев Жемчужниковых и А.К. Толстого. Автор: Олег Христенко Вопрос 15: Вспомним "Даму с горностаем". Википедия твердит, Мол, совсем не горностая Леонардо рисовал. По одной из новых версий, Этот зверь - один из НИХ. В общем, ИХ двумя словами Мы попросим вас назвать. Ответ: Белые хори. Зачет: Белые хорьки. Комментарий: Если верить Википедии, на картине изображен фуро - белый хорь, а вовсе не горностай. А вопрос, чтобы вам подсказать, написан белым хореем. Источник: http://ru.wikipedia.org/wiki/Хорьки Автор: Борис Гуревич Тур: 5 тур. Команда Павла Ершова Дата: 24-Mar-2012 Редактор: Павел Ершов Вопрос 1: Эпиграф. "70% людей находят пошлость в каждом предложении". В это высказывание из Интернета мы внесли незначительное изменение. Восстановите исходный вариант. Ответ: "69% людей...". Автор: Павел Ершов Вопрос 2: "Видео"-дуплет. (pic: 20110726.jpg) 1. Эта божественная скульптура Праксителя носит название "Гермес с маленьким ...нисом". Вставьте пропущенные в названии буквы. (pic: 20110727.jpg) 2. На этом портрете вы можете обнаружить очертания ...ениса Давыдова, но вопрос, по просьбе тестеров, не ПРО ЭТО. Перед вами портрет графа Давыдова работы Кипренского. Какие две буквы мы пропустили в предыдущем предложении? Ответ: 1. Дио. 2. Ев. Комментарий: 1. Адонис и Дафнис не боги и вообще не доросли. 2. На картине изображен Евграф Давыдов, родственник Дениса, черты которого, несмотря на протесты тестеров, легко можно обнаружить во внешности Евграфа. Если вы всё сделали правильно, то из пропущенных букв смогли составить слово "видео". Источник: 1. http://old.marin.ru/greece/peloponnese_excursions.shtml 2. http://ru.wikipedia.org/wiki/Давыдов,_Евграф_Владимирович Автор: Павел Ершов Вопрос 3: Дуплет. 1. По легенде Кронос обескуражил Урана, оскопив его. А чем Кронос был вооружен? 2. По легенде Керенский покинул Зимний дворец в женском платье. А лет двадцать спустя в женском платье видели Троцкого, что задержало отправление "Его и Ее" за границу. Назовите "Его и Ее". Ответ: 1. Серп. 2. "Рабочий и Колхозница". Комментарий: Для чистоты, да что там - стерильности эксперимента мы решили начать не с яиц, а с серпов. 1. Выражение "серпом по яйцам" означает обескураживающее событие, приводящее к полной дезориентации и отчаянию. 2. Директор завода, где шла работа над монументом, углядел черты Троцкого в складках юбки колхозницы. Только после пристальной проверки скульптуру пустили на Парижскую Выставку. А еще по одной легенде, выражение "серпом по яйцам" возникло в результате ссоры рабочего и колхозницы. Источник: 1. http://myths.kulichki.ru/enc/item/f00/s35/a003557.shtml 2. http://www.gazeta.lv/story/8656.html 3. http://www.slovoborg.ru/definition/серпом+по+яйцам Автор: Павел Ершов Вопрос 4: [Ведущему: "Нечто неизвестное" прочитать, усердно подчеркивая средний род.] Лишний раз напомним, что ИКС в вопросе заменяет нечто неизвестное. Если следовать логике Александра Гениса, ИКС - Эрастович. Назовите ИКС двумя словами, начинающимися на одну букву. Ответ: Отчество Онегина. Комментарий: Анализируя "Бедную Лизу", Генис (на пару с Вайлем) допускает, что Эраст - "открывающий галерею лишних людей русской литературы" - мог быть отцом Онегина. У Пушкина отчество Евгения не упоминается. Это последний вопрос об "енисах". Источник: http://lib.ru/PROZA/WAJLGENIS/literatura.txt_with-big-pictures.html Автор: Павел Ершов Вопрос 5: В романе Герта Хофмана шестеро мужчин подряжаются на работу и дружно плетутся к дому заказчика. Мы не спрашиваем, кто был у них за Старшего. Назовите трехбуквенным словом место, где все они, в полном соответствии с известным выражением, оказываются в конце концов. Ответ: Яма. Зачет: Ров. Комментарий: "Если слепой ведет слепого, то оба они упадут в яму". Хофман воссоздает историю создания картины Брейгеля Старшего "Притча о слепых". Источник: http://en.wikipedia.org/wiki/Der_Blindensturz Автор: Павел Ершов Вопрос 6: Внимание, ЧГК наоборот! Запишите, пожалуйста, ответ: "Беллона Марсу". А теперь зачет: "Сехмет Нефертуму". Через минуту напишите вопрос, с точностью до знака совпадающий с поговоркой, а для кого-то, может, и пословицей. Ответ: Кому война, а кому мать родна. Комментарий: Приведены богини войны в разных мифологиях и их дети. Источник: 1. Энциклопедия "Кругосвет", ст. "Беллона". 2. http://ru.wikipedia.org/wiki/Нефертум Автор: Павел Ершов Вопрос 7: Цитата: "Вениамин Каверин тоже еврей. Правда, из Харькова". К этой цитате литературный редактор делает примечание: "На самом деле Каверин из Пскова и автор наверняка знал это". Назовите этого автора двумя словами, не начинающимися на одну букву. Ответ: Сергей Довлатов. Зачет: Довлатов Сергей. Комментарий: "... Но не желал в одной фразе употреблять дважды букву "П": "Правда, из Пскова". (Ред.) Довлатов, как известно автору вопроса и неизвестно тестерам, избегал использования двух слов на одну букву в одном предложении. Источник: С. Довлатов. Литература продолжается. Автор: Павел Ершов Вопрос 8: Говорят, еще перед выходом на Поле в сентябре 80-го, Александр знал о превосходстве соперника по одному показателю. Поэтому Александр решил отказаться от защиты и в итоге не проиграл. Назовите соперника Александра. Ответ: Челубей. Комментарий: Копье Челубея было длиннее стандартного, что не позволяло сопернику нанести удар. Пересвет избавился от кольчуги, чтобы быть пронзенным, но не выпасть из седла. Это позволило ему, будучи смертельно раненным, не только поразить противника, но и вернуться в строй верхом. Источник: http://ru.wikipedia.org/wiki/Александр_Пересвет Автор: Павел Ершов Вопрос 9: Вера Герберта Уэллса в свободную любовь и сексуальное раскрепощение сделала его кем-то вроде покровителя для последующих поколений. Иллюстрацией этому может служить тот факт, что Уэллс очутился однажды во втором ряду... Где? Ответ: На обложке "Сержанта Пеппера". Зачет: Ответ, из которого ясно, что речь идет об обложке альбома "Sgt. Pepper's Lonely Hearts Club Band". Комментарий: Между Марксом и Йогонандой. Словами "покровитель" и "иллюстрация" мы пытались навести на обложку, "свободной любовью" - на хиппи, "вторым рядом" - на второй ряд. Источник: 1. Р. Шнакенберг. Тайная жизнь великих писателей. - М.: Книжный клуб 36.6, 2010. - С. 166. 2. http://math.mercyhurst.edu/~griff/sgtpepper/people.html Автор: Павел Ершов Вопрос 10: <раздатка> Оди из поседних сучаев ЕЕ приенения афиксирован в 918 году и поучил названи "лотерея Хууслахти". Назовите ЕЕ. Ответ: Децимация. Зачет: По смыслу. Комментарий: Во время Гражданской войны в Финляндии зафиксирован случай "выборочного" расстрела 80 пленных красногвардейцев. Исходную фразу "Один из последних случаев ЕЕ применения зафиксирован в 1918 году и получил название "лотерея Хуруслахти"" мы, в соответствии с римским пониманием децимации, разбили на десятки и в каждом произвольным образом избавились от одного символа. Источник: http://www.torturesru.com/dle/execution/374-decim225ciya.html Автор: Павел Ершов Вопрос 11: В этюде отечественного шахматиста Петрова белые кони преследуют черного короля. Что изображает в этой задаче восьмиклеточная белопольная диагональ? Ответ: Березина. Комментарий: В этюде по мотивам Отечественной войны "Бегство Наполеона из Москвы в Париж" восьмиклеточная белая диагональ обозначает восьмибуквенную заснеженную реку. Источник: http://www.chess-club-slon.ru/art_petrov.php Автор: Павел Ершов Вопрос 12: Нарекая своего героя этим именем, Чуковский, возможно, имел в виду фразу, которую мог бы произносить герой. Воспроизведите эту фразу-палиндром. Ответ: Карудо - дурак! Комментарий: Сам Чуковский утверждал, что словом "карудо" называла соседского попугая его дочь Мура. Источник: К. Чуковский. Доктор Айболит (любое уважающее себя издание). Автор: Павел Ершов Вопрос 13: (pic: 20110728.jpg) Замок Трутц Эльтц был построен в сжатые сроки с весьма прагматичной целью. Назовите эту цель максимально точно. Ответ: Осада замка Эльтц. Зачет: Взятие замка Эльтц и т.п. по смыслу. Комментарий: На розданном фото - вид на замок Эльтц со стен так называемого контр-замка Трутц Эльтц (от нем. trutz - бой, борьба, а также уст. вопреки). Обычным способом осады было строительство второй крепости по соседству, с которой велась бомбардировка. Источник: 1. http://www.burg-eltz.de/e_geschichte_fehde.html 2. http://www.mir-zamkov.net/rp/eltz/o.shtml 3. http://commons.wikimedia.org/wiki/File:Burg_Eltz_und_Burg_Trutzeltz.jpg Автор: Павел Ершов Вопрос 14: (pic: 20110729.jpg) В 1941 году в эту церковь попала бомба. В результате, можно сказать, в течение двадцати лет на земле не появилось ни одного истинного ЗАМОЧИ. Какое слово мы заменили на "ЗАМОЧИ"? Ответ: Кокни. Комментарий: Истинный кокни должен родиться в пределах слышимости колоколов Сент-Мэри-ле-Боу. Колокола церкви зазвучали после реставрации лишь в 1961 году. Источник: http://ru.wikipedia.org/wiki/Кокни Автор: Павел Ершов Вопрос 15: Просьба командам от ласточек: поднимайте ваши карточки выше. В конкурсе 2010 года, обойдя "Stairway to Heaven" и ряд других зажигательных баллад, победила "Богемская рапсодия" "Queen". Назовите компанию, которая организовала этот конкурс. Ответ: "Zippo". Комментарий: Был составлен рейтинг песен, под которые лучше всего качаться с зажженной зажигалкой в руке. И никаких яблок. Источник: http://www.zippo.ru/news/id-28/ Автор: Павел Ершов Тур: 6 тур. Команда Сергея Дорофеева Дата: 24-Mar-2012 Редактор: Александр Коробейников Инфо: Команда благодарит за тестирование команду "МИД-2" и сборную Скайпа, а также Михаила Иванова, Бориса Моносова, Евгения Поникарова, Александра Чижова. Вопрос 1: Талисманы спортивных турниров преимущественно представляют животный мир. Заголовок статьи о них на пару букв длиннее названия известного произведения середины XX века. Напишите название статьи, состоящее из двух слов. Ответ: "Маскотный двор". Зачет: "Маскотское хозяйство", "Маскотная ферма". Комментарий: Талисманы еще называются маскотами. Источник: http://ekaterinburg.bezformata.ru/listnews/talismani-dlya-sportivnih-komand/527140/ Автор: Сергей Дорофеев Вопрос 2: Жители окрестностей КАтве, что в Уганде, совсем не любят дождь. Говоря о российском аналоге КАтве, "Книга Большому Чертежу" упоминает лед. Назовите этот аналог. Ответ: Баскунчак. Комментарий: Катве - это соляное озеро-месторождение. Местные жители не любят дождь, так как он делает соль менее чистой и концентрированной. В Росси крупнейшим соляным озером-месторождением является Баскунчак, где "ломают соль чистую, как лед". Источник: 1. "Вокруг света", 2011, N 3. - С. 38. 2. http://www.salz.ru/baskunchak.html Автор: Александр Коробейников Вопрос 3: В классических азербайджанских и турецких комедиях героини и субретки часто носят имя ТеллИ - "кудрявая". Назовите профессию человека, которого героиня одной такой комедии прозвала "Три кудряшки". Ответ: Учитель. Зачет: Учитель танцев. Комментарий: "Уч" - три на большинстве тюркских языков (вспомните "Учкудук - три колодца"). Героиня называла так приглашенного русского учителя. Источник: 1. Л.М. Вайсенберг. Младшая сестра. 2. У. Гаджибеков. Аршин мал алан. 3. Ю. Гусман. Сценарий "Не бойся, я с тобой". Автор: Галина Буланова Вопрос 4: Говоря об ИКСЕ в ИГРЕКЕ, российские болельщики обычно представляют себе крупный сибирский город. Кстати, ИКС - фамилия серебряного призера Олимпийских игр 1900 года в составе сборной Великобритании по ИГРЕКУ. Назовите ИКС и ИГРЕК. Ответ: Дерби, Регби. Комментарий: Самое крупное дерби в регби для России - это матч "Енисей-СТМ" - "Красный Яр". Обе команды представляют Красноярск. Источник: http://ru.wikipedia.org/wiki/Дерби,_Артур Автор: Сергей Дорофеев Вопрос 5: Астрономы считают, что ОНО включает в себя многочисленные астероиды класса V [вэ]. Согласно же мифологии, ОНО не включало в себя детей. Назовите ЕГО двумя словами. Ответ: Семейство Весты. Комментарий: Веста - крупный астероид и целомудренная богиня. Источник: http://ru.wikipedia.org/wiki/Семейство_Весты Автор: Сергей Дорофеев Вопрос 6: Герой советской песни возвращается в родную деревню. Цитата: Глаза его хозяйские осматривают рожь, Шумит она, красавица, звенят-поют ИКСЫ, И парень улыбается в ИГРЕЧНЫЕ усы. Какие слова мы заменили словами "ИКСЫ" и "ИГРЕЧНЫЕ"? Ответ: Овсы, пшеничные. Комментарий: Неудивительно, что тема зерновых проходит красной нитью через песню. Источник: http://www.sovmusic.ru/text.php?fname=poyotga Автор: Галина Буланова Вопрос 7: <раздатка> Как [пропуск] и сыро, И впрямь, осенняя пора... Ну, слава Богу, вот и Выра, Огни знакомого двора! В розданном вам четверостишии Валентина Вихорева мы пропустили начало другого известного произведения. Заполните пропуск. Ответ: "... нынче ветрено...". Комментарий: И волны с перехлёстом. Источник: http://www.peoples.ru/art/music/national/valentin_vihorev/vyra.shtml Автор: Сергей Дорофеев Вопрос 8: Внимание, в вопросе глагол "ПРОИЗНОСИТЬ" заменяет другой глагол. Герой Юрия Коваля произносил: "А клада нам не надо!". Какую нетипичную пару произносил заглавный герой советского произведения? Ответ: Палка, селедка. Комментарий: Произносить = рифмовать. Источник: 1. Ю. Коваль. Суер-Выер. 2. Н. Носов. Приключения Незнайки и его друзей. Автор: Сергей Дорофеев Вопрос 9: В биографическом фильме режиссера Мануэля Эрреры главный герой, его соотечественник, приезжает в заснеженную Москву, где ему говорят, что в Москве очень жарко из-за НЕЕ. Назовите ЕЕ двумя словами. Ответ: Шахматная горячка. Зачет: Шахматная лихорадка. Комментарий: Эррера - кубинец, как и Хосе Рауль Капабланка, являющийся героем этого фильма. Капабланка снялся в фильме "Шахматная горячка". Источник: Х/ф "Капабланка", реж. М. Эррера, 1986 г. Автор: Александр Коробейников Вопрос 10: Александр Гаррос сравнивает этих людей с искателями Святого Грааля, не обращавшими внимания на войны и революции вокруг. По его мнению, неудивительно, что один из них впоследствии стал высшим моральным авторитетом и носителем милосердия. Назовите этого человека. Ответ: Фритьоф Нансен. Комментарий: Гаррос пишет о полярных исследователях. Один из них, Нансен, впоследствии стал комиссаром Лиги Наций по делам беженцев. Источник: http://www.asonov.com/vokrug-sveta/vokrug-sveta/bremja-belogo.html Автор: Александр Коробейников Вопрос 11: В стихотворении Семена Кирсанова говорится: "Пляшут никель, железо, кальций / С ускорением в тыщу раз". Далее поясняется, что это танец ИХ. Поняв, где происходит дело, назовите ИХ словом, заимствованным из немецкого языка. Ответ: Протуберанцы. Комментарий: Стихотворение называется "Танцевальный час на Солнце". Источник: http://www.alhimik.ru/fun/stihochem5.html Автор: Галина Буланова Вопрос 12: В Западной Африке профессиональные ИКСЫ носят с собой небольшие фигурки, чтобы чего-нибудь не пропустить. Какому ИКСУ установлен памятник в саду Тюильри? Ответ: Шарлю Перро. Комментарий: Сказочники, чтобы не забыть всех персонажей сказок, носят с собой их фигурки. Источник: 1. А.А. Леонтьев. Путешествие по карте языков мира. http://www.flibusta.net/b/222299/read 2. http://www.nkj.ru/archive/articles/4897/ Автор: Александр Коробейников Вопрос 13: Из маркетинговых соображений ИКС в музее ратуши Стокгольма - единственный. "ИКС" - название документального фильма. Назовите ИКС четырьмя словами. Ответ: "Выход через сувенирную лавку". Комментарий: Чтобы туристы лишний раз подумали, не приобрести ли им сувениров. Фильм Бэнкси о самом Бэнкси и его коллегах. Источник: 1. ЛОАВ. 2. http://ru.wikipedia.org/wiki/Выход_через_сувенирную_лавку Автор: Александр Коробейников Вопрос 14: Заголовок статьи о том, что сборная Чехии по теннису выиграла Кубок Федерации большей частью благодаря своему лидеру, лишь на одну букву отличается от названия посуды. Воспроизведите этот заголовок из двух слов. Ответ: "Чашка Петры". Источник: http://sport.rambler.ru/news/tennis/591058635.html Автор: Григорий Головин Вопрос 15: В конце произведения Сергея Лукьяненко Юрий Гагарин окидывает взглядом картину космического побоища и со всем накопившимся чувством говорит... Какое слово? Ответ: Приехали. Комментарий: Начал с "поехали"... Источник: С. Лукьяненко. Лунный Рейх. Автор: Сергей Дорофеев Тур: 7 тур. Команда Дмитрия Карякина Дата: 24-Mar-2012 Редактор: Антон Снятковский, Дмитрий Карякин, Константин Смолий, Денис Лагутин Инфо: Редакторы выражают благодарность за ценные замечания, высказанные при подготовке пакета, Александру Коробейникову, Михаилу Перлину и Алине Малиновской. Вопрос 1: [Нулевой вопрос] По мнению автора вопроса, если бы талисман компании "Michelin" был разработан студией Артемия Лебедева, он тоже мог бы выполнять свою функцию, но назывался бы не Бибендум, а ИКС. Назовите двумя словами то, неформальным названием чего является ИКС. Ответ: Гиперболический синус. Комментарий: Товары студии Лебедева принято называть псевдолатинскими словами (лабиринтус, пазлус). Составленный из шин человечек - символ компании "Michelin" - вполне мог бы называться шинус. Шинус и чосинус - неофициальные названия гиперболических синуса и косинуса, обозначающихся sh(x) и ch(x). Этим вопросом автор передает привет Евгению Пашковскому, который однажды сказал, что закончит играть в ЧГК, если ему придется играть вопрос с ответом "гиперболический косинус". Однако поскольку такого ответа у нас пока всё же не прозвучало, мы выражаем надежду на то, что Евгений продолжит свои выступления, в том числе и за нашу команду. Источник: 1. http://store.artlebedev.ru/ 2. http://en.wikipedia.org/wiki/Bibendum 3. http://ru.math.wikia.com/wiki/Гиперболические_функции Автор: Дмитрий Карякин Вопрос 2: В современном рассказе по мотивам биографии Фриденсрайха Хундертвассера описывается его путешествие из Баварии в Австрию с кинорежиссером Петером Шамони. Во время этого путешествия Хундертвассер замечает, что, отбросив патриотизм, предпочел бы отдыхать в ПЕРВОМ, а не во ВТОРОМ. Рассмеявшись, Шамони отвечает, что ничего другого от Фриденсрайха Хундертвассера он и не ожидал. Назовите ПЕРВЫЙ и ВТОРОЙ. Ответ: Гармиш-Партенкирхен, Инсбрук. Зачет: В любом порядке. Комментарий: Петера развеселило то, что Фриденсрайх Хундертвассер предпочел бы австрийскому Инсбруку находящийся неподалеку немецкий трудновыговариваемый Гармиш-Партенкирхен. Оба населенных пункта являются горнолыжными курортами и в свое время были столицами Зимних Олимпийских игр. Забавно, что французский Шамони (Chamonix), название которого созвучно с фамилией спутника Хундертвассера (Schamoni), также является и тем, и другим. Источник: 1. http://www.proza.ru/2012/03/24/725 2. http://www.hundertwasser.ru/biography/index.htm Автор: Андрей Ефимцов Вопрос 3: Нулевой вопрос, задававшийся в качестве преамбулы к данному туру, редактор пакета сравнил с НИМИ. При ИХ установке автору вопроса случалось пользоваться числами от единицы до восьмерки. Места, где в большом количестве можно увидеть ИХ, легко найти как в Инсбруке, так и в Гармиш-Партенкирхене. Назовите ИХ двумя словами. Ответ: Красные флажки. Зачет: Красные флаги. Комментарий: В игре "Сапер" красные флажки обозначают предполагаемое игроком расположение мины. На слаломных трассах (которые легко найти в указанных городах) полным-полно красных флажков. Нулевой вопрос как бы предупреждает играющих, что их ждет не самый легкий тур. Источник: ЛОАВ. Автор: Антон Снятковский Вопрос 4: СССР появилась в феврале 1921 года, а в ноябре 1990 года она была переименована. Как расшифровывается первая буква в этой аббревиатуре? Ответ: Сакартвелос. Зачет: Сакартвело. Комментарий: Речь идет о Грузинской Советской Социалистической Республике (Сакартвелос Сабчота Социалистури Республика). Источник: 1. http://ru.wikipedia.org/wiki/Грузинская_Советская_Социалистическая_Республика 2. http://www.mining-enc.ru/g/gruzinskaya-sovetskaya-socialisticheskaya-respublika/ Автор: Антон Снятковский Вопрос 5: Героиня романа Касслера Клайва безуспешно пытается сделать логический вывод на основе фактов, между которыми не прослеживается никакой связи, после чего вспоминает о смерти одной женщины. Назовите профессию родственника этой женщины, получившего определенную известность в первой четверти XX века. Ответ: Швейцар. Комментарий: Героиня уподобляет свои размышления попытке ответить на известную загадку солдата Швейка: "Стоит четырехэтажный дом, в каждом этаже по восьми окон, на крыше - два слуховых окна и две трубы, в каждом этаже по два квартиранта. А теперь скажите, господа: в каком году умерла у швейцара бабушка?". Роман "Похождения бравого солдата Швейка во время мировой войны" был написан и опубликован в первой четверти XX века. Источник: 1. Касслер Клайв. Змей. http://www.litmir.net/br/?b=13670&p=79 2. Ярослав Гашек. Похождения бравого солдата Швейка. Автор: Иван Веселов Вопрос 6: (pic: 20110730.jpg) Назовите британский город, в котором происходит большая часть действия фильма, кадр из которого мы вам раздали. Ответ: Манчестер. Комментарий: На розданном кадре Мэтт Басби прощается с командой, погибшей в авиакатастрофе под Мюнхеном. Гробы расположены в соответствии с футбольной расстановкой 3-4-3, а самого Басби можно принять за вратаря. Кстати, голкипер "Манчестер Юнайтед" в той авиакатастрофе выжил. z-checkdb: На розданном кадре не тренер Мэтт Басби, а его ассистент Джимми Мерфи (Александр Рождествин). Источник: 1. Х/ф "Юнайтед", 2011. 2. http://www.imdb.com/title/tt1777034/ Автор: Денис Лагутин Вопрос 7: В этом вопросе имена Эдвард и Альберт являются заменами. В 1989 году кавалер ордена "Крест Виктории" Эдвард, полагавший, что народ в нем нуждается, попытался вернуться в страну. Готовясь к восстановлению власти, он прибыл в столицу соседнего государства, однако через некоторое время был отправлен назад местным правителем Альбертом. Каких двух людей мы заменили Эдвардом и Альбертом? Ответ: Иди Амин и Мобуту Сесе Секо. Комментарий: Три озера в восточной Африке были названы в честь королевы Виктории, ее мужа Альберта и их старшего сына Эдуарда VIII. Два из них были переименованы диктаторами соседних стран и некоторое время назывались Иди-Амин-Дада (Эдвард) и Мобуту-Сесе-Секо (Альберт). Мобуту отправил Амина назад в Саудовскую Аравию. Источник: http://ru.wikipedia.org/wiki/Амин,_Иди Автор: Дмитрий Карякин Вопрос 8: (pic: 20110731.jpg) На розданном вам изображении, согласно его назначению, в частности, ДЕЛАЮТ ЭТО. ЭТО ДЕЛАЮТ обычно перед тем, как кормят птиц. Какое словосочетание мы заменили словами "ДЕЛАЮТ ЭТО"? Ответ: Раскрашивают хлеб. Зачет: Раскрашивают булку, батон, каравай и т.д. Комментарий: Слово "раскрашивать" является также несовершенным видом глагола "раскрошить". Источник: 1. http://www.deti-lit.ru/files/0/115769641253749976.jpg 2. http://dic.academic.ru/dic.nsf/ushakov/1001406/ Автор: Михаил Перлин Вопрос 9: В книге Терри Пратчетта кошка, отличавшаяся особой подозрительностью, дышала с таким выражением на мордочке, как будто даже ОН был ТАКИМ. В произведении 1930 года слова "ТАКОЙ ОН", по всей вероятности, относятся и к самому этому произведению. Какие два слова, начинающиеся на одну и ту же букву, мы заменили словами "ТАКОЙ ОН"? Ответ: Ворованный воздух. Комментарий: В книге "Четвертая проза" Осип Мандельштам сравнивает написанную без разрешения литературу с ворованным воздухом. Источник: 1. Т. Пратчетт. Кот без прикрас. - М.: Эксмо, 2010. - С. 120. 2. http://chernov-trezin.narod.ru/TitulSholohov.htm Автор: Антон Снятковский Вопрос 10: К названию деревни, которую можно найти юго-западнее "острова", расположенного в центре страны, в 1971 году было добавлено название вымышленного города, впервые описанного примерно семьюстами сорока одним месяцем ранее. При проведении расчетов мы немного потеряли в точности, не учитывая дни, часы и минуты. Назовите того, кто придумал этот вымышленный город. Ответ: Марсель Пруст. Комментарий: Деревня Илье, расположенная рядом с областью Иль-де-Франс, считается прототипом города Комбре из гепталогии "В поисках утраченного времени" Марселя Пруста и была переименована в Илье-Комбре в год столетия со дня рождения писателя. Источник: 1. http://fr.wikipedia.org/wiki/Illiers-Combray 2. http://fr.wikipedia.org/wiki/%C3%8Ele-de-France 3. http://fr.wikipedia.org/wiki/Du_c%C3%B4t%C3%A9_de_chez_Swann Автор: Дмитрий Карякин Вопрос 11: [Ведущему: кавычки не озвучиваются.] Викунья - южноамериканское животное, а глициния - растение, гораздо более распространенное в Японии, чем в Южной Америке, однако важную роль в деле восстановления популяции викуний сыграл "Глициниевый лес". А как звучат слова "Глициниевый лес" по-японски? Ответ: Фудзимори. Зачет: Фухимори. Комментарий: При Альберто Фухимори (президент Перу в 1990-2000) был принят Закон о викунье, в результате чего это животное было взято под защиту государства. Фухимори - этнический японец. Источник: 1. http://www.vicuna.ru/index.php/historia-de-proteccion/sistema-nacional/ 2. http://www.vicuna.ru/index.php/historia-de-proteccion/semicautiverio/ 3. http://en.wikipedia.org/wiki/Fujimori Автор: Антон Снятковский Вопрос 12: Болельщики вспоминают, что поле в тот день было раскисшее, а мяч тяжелый и скользкий, и Алексей Хомич сказал ЕМУ: "Опасно играешь". Мы не спрашиваем, что произошло через несколько минут. Назовите ЕГО. Ответ: Леонид Шмуц. Комментарий: Голкипер ЦСКА Леонид Шмуц вошел в историю в 1971 году вследствие удивительного автогола, когда он в матче с "Араратом" при вводе мяча в игру рукой забросил его в собственные ворота. Знаменитый некогда вратарь Хомич к тому времени переквалифицировался в фотокоры. Он стоял за воротами и благодаря своему опыту смог предвосхитить такое необычное развитие событий. Источник: http://forum.1tv.ru/lofiversion/index.php/t64959-0.html Автор: Антон Снятковский Вопрос 13: Внимание, в вопросе слова "ПРИСТЯЖНЫЕ" и "КОРЕННЫЕ" являются заменами. Вадим Наумов пишет, что словом "самоеды" в прошлом называли ненцев, энцев, а также другие ПРИСТЯЖНЫЕ КОРЕННЫЕ народы Севера. Какое четырнадцатибуквенное слово мы заменили словом "ПРИСТЯЖНЫЕ"? Ответ: Многочисленные. Комментарий: А словом "КОРЕННЫЕ" - слово "малочисленные". Источник: http://m-yu-sokolov.livejournal.com/2044068.html Автор: Антон Снятковский Вопрос 14: В предисловии к книге "Язык командорских алеутов" ее авторы благодарят тех, кто смог обогатить их уникальными материалами по вымирающему языку, и грустно шутят, что более точно эту книгу следовало бы назвать чуть иначе. Как именно? Ответ: "Язык командорских алеуток". Комментарий: Как это часто бывает, вымирающий язык помнят лишь немногочисленные женщины преклонного возраста. Источник: Е.В. Головко, Н.Б. Вахтин, А.С. Асиновский. Язык командорских алеутов. - СПб.: Наука, 2009. - С. 6. Автор: Антон Снятковский Вопрос 15: По всей вероятности, один из предков этого человека был весьма красив, а сам он в середине XX века доказал одну из самых красивых теорем. Брат этого человека был орнитологом и проводил исследования на территории современной Демократической Республики Конго. Напишите фамилию этого человека. Ответ: Пригожин. Комментарий: Демократическая Республика Конго (она же Заир) до 1960 года была бельгийской колонией. Физик и химик Илья Романович Пригожин, родившийся в Российской империи, прославился уже как бельгийский ученый и получил Нобелевскую премию по химии в 1977 году. Его брат Александр получил известность как орнитолог; неудивительно, что работал он именно в Бельгийском Конго. Илья Пригожин в 1947 году доказал теорему о возможности самозарождения порядка из хаоса. Старинное прилагательное "пригожий" имеет значение "красивый, миловидный". Источник: 1. http://ru.wikipedia.org/wiki/Пригожин,_Илья_Романович 2. http://www.ufolog.ru/names/order/Пригожин Автор: Дмитрий Карякин и Антон Снятковский Вопрос 16: Внимание, в вопросе слова "ТАКОЙ" и "ИКС" являются заменами. Казнь, при которой на спине осужденного рассекали ребра, разводили их в стороны и вытаскивали наружу легкие, называлась "ТАКОЙ ИКС". В произведении 1822 года упоминаются ИКС и ТАКАЯ пища. Назовите это произведение. Ответ: "Узник". Зачет: "Сижу за решеткой в темнице сырой...". Комментарий: Казнь эта называлась "Кровавый орел". Источник: 1. http://ru.wikipedia.org/wiki/Кровавый_орёл 2. http://www.feb-web.ru/feb/pushkin/texts/push10/v02/d02-120-.htm Автор: Дмитрий Карякин Тур: 8 тур. Команда Анны Рогозиной Дата: 24-Mar-2012 Инфо: Команда благодарит за тестирование пакета и ценные замечания Михаила Локшина и команду "Юма", а также Мишеля Матвеева (все - Санкт-Петербург). Вопрос 1: Согласно Википедии, ИХ лидером является гвардии полковник Дятел. Назовите ИХ одним словом. Ответ: "Стрижи". Комментарий: Авиационная группа высшего пилотажа. Источник: http://ru.wikipedia.org/wiki/Дятел,_Николай_Михайлович Автор: Анна Рогозина Вопрос 2: Гуляя по ВДНХ, автор вопроса увидел вывеску, слово на которой начиналось на "М" и кончалось на "НЯ". Восстановите восемь пропущенных букв. Ответ: ацепекар. Источник: ЛОАВ. Автор: Игорь Доскоч Вопрос 3: Православная журналистка Надежда Локтева, рассуждая о литературе в жанре фэнтези, заметила, что разница между сказочными волшебниками и реальными колдунами такая же, как между ПЕРВЫМИ И ВТОРЫМИ. ВТОРЫХ было примерно в 666 раз больше, чем ПЕРВЫХ. Назовите ПЕРВЫХ и ВТОРЫХ максимально точно. Ответ: Три поросенка и гадаринские свиньи. Комментарий: Если верить евангелисту Марку, то гадаринских свиней было около двух тысяч. По мнению Локтевой, сказочные волшебники вполне безобидны, в отличие от колдунов, одержимых бесами. Третий вопрос. :-) Источник: А. Кураев. Гарри Поттер в церкви: между анафемой и улыбкой. Автор: Виталий Федоров Вопрос 4: Ученые полагают, что со временем спутник, называющийся ИКС, может изменить форму. В эпизоде под названием "ИКС" известного произведении пес последовательно принимает образы зайца, оленя, медведя, теленка, леопарда. Назовите имя собственное, которое мы заменили на "ИКС". Ответ: Протей. Комментарий: Морское божество Протей могло принимать любой облик. Спутник Нептуна Протей - самый несферический из спутников, но это дело времени. Произведение - "Улисс" Джойса, соответствующая глава. Источник: 1. http://www.sciencedirect.com/science/article/pii/0019103592901562 2. Дж. Джойс. Улисс. Автор: Игорь Биткин Вопрос 5: Гуманитарный вопрос. Согласно философу Мережковскому, существует еще и "вместилище Божественного Духа". Литературовед Борис Соколов пишет, что Булгаков, развивая мысль Мережковского, "изобретает" ЭТО - обитель дьявола. Назовите ЭТО двумя словами. Ответ: Пятое измерение. Комментарий: По Мережковскому есть и четвертое измерение. Пятый вопрос. :-) Источник: 1. http://www.litmir.net/br/?b=114010&p=6 2. Б. Соколов. Расшифрованный Булгаков. Автор: Виталий Федоров Вопрос 6: Свой первый патент этот изобретатель получил в 1831 году - на сеялку, которая разбрасывала зерна риса кучно и равномерно. Назовите его фамилию. Ответ: Гатлинг. Комментарий: Пулемет тоже разбрасывает пули кучно и равномерно. :-) Источник: 1. http://ru.wikipedia.org/wiki/Гатлинг,_Ричард_Джордан 2. http://shkolazhizni.ru/archive/0/n-10124/ Автор: Виталий Федоров Вопрос 7: Ричард Фейнман, упомянув дефицит национального бюджета США, сказал, что теперь ИХ следовало бы называть экономическими. Назовите ИХ двумя словами. Ответ: Астрономические числа. Источник: Р. Фейнман. Дюжина лекций. Автор: Виталий Федоров Вопрос 8: Наум Синдаловский вспоминает такую байку. Правила кафе запрещали курение внутри помещения, поэтому посетители выходили курить в тесный коридор, который наполнялся густыми облаками дыма. Однажды подошел милиционер и сказал: "Безобразие, какой-то ИКС устроили". Напишите современное название ИКСА. Ответ: Хошимин. Комментарий: Таково, по одной из версий, происхождение неофициального названия знаменитого ленинградского кафе "Сайгон". Источник: 1. Н.А. Синдаловский. Пороки и соблазны северной столицы. 2. http://ru.wikipedia.org/wiki/Хошимин Автор: Виталий Федоров Вопрос 9: (pic: 20110732.jpg) Назовите фамилию человека, в рабочем кабинете которого, если верить журналисту Роману Вагину, висит этот плакат. Ответ: Смородская. Комментарий: Плакат "Женщина - на паровоз" якобы висел в кабинете президента ФК "Локомотив". Источник: Эфир "Радио Спорт", 20.04.2011 г. Автор: Виталий Федоров Вопрос 10: (pic: 20110733.jpg) Перед вами рекламный плакат некой организации. Подпись под ним гласит: "Если вы [ПРОПУСК], то вы знаете, куда обратиться". Заполните пропуск. Ответ: "... видите две буквы...". Зачет: По смыслу. Комментарий: Общество Анонимных Алкоголиков. Автор: Виталий Федоров Вопрос 11: В XIX веке из примерно девятисот ИХ более семисот принадлежали США. Они обеспечивали страну горюче-смазочными материалами, сырьем для лакокрасочной, легкой и пищевой промышленности. Назовите ИХ максимально точно. Ответ: Китобойные суда. Источник: С. Левитт, С. Дабнер. Суперфрикономика. Автор: Виталий Федоров Вопрос 12: Говоря о мирном сосуществовании религий в Советском Союзе, Михаил Задорнов упоминает четыре символа. Назовите их все в любом порядке. Ответ: Серп, молот, крест, полумесяц. Комментарий: Символы креста и полумесяца похожи на молот и серп. После развала СССР на его территории начались конфликты христиан и мусульман. Источник: Концерт на канале "Рен-ТВ", 24.12.2011 г. Автор: Виталий Федоров Вопрос 13: В современной экранизации Шерлок Холмс узнает о планируемом теракте, который спецслужбы не собираются предотвращать. Поняв, в чем дело, он произносит одно слово, на что его собеседница отвечает, что никогда там не была. Где? Ответ: В Ковентри. Комментарий: Известна история, возможно легендарная, что англичане заранее узнали о планах бомбардировки Ковентри, но не стали принимать никаких специальных мер, чтобы не раскрыть источник информации. Источник: Сериал "Шерлок". Автор: Виталий Федоров Вопрос 14: Известный современный ученый считает, что вопрос: "Вы какой ИКС, католический или протестантский?" вполне правомерен. Он же замечает, что ИКСЫ в современной Америке подвергаются такой же дискриминации, как гомосексуалисты полвека назад. Назовите ИКСОВ. Ответ: Атеисты. Комментарий: Даже атеисты, выросшие в определенной религиозной культуре, отличаются друг от друга. В США атеистов несколько недолюбливают. Источник: Р. Докинз. Бог как иллюзия. Автор: Виталий Федоров Вопрос 15: Выступая на теледебатах, Жириновский сказал: Я такой, какой я есть, в этом [ПРОПУСК]. На сайте либ.ру в книге, увидевшей свет в 1937 году, пропуск встречается неоднократно, причем в первом случае в нем три буквы "с". Заполните пропуск двумя словами. Ответ: Моя прелесть. Комментарий: Дж.Р.Р. Толкин. Хоббит. Автор: Анна Рогозина Тур: 9 тур. Команда Михаила Савченкова Дата: 07-Apr-2012 Инфо: Команда благодарит за тестирование и ценные советы при подготовке пакета Вадима Молдавского (Лондон) и Сергея Николенко (Санкт-Петербург). Вопрос 1: Известный политик говорит, что всю жизнь боролся с привычкой раздувать щеки. Назовите этого политика. Ответ: Билл Клинтон. Комментарий: Широко известно его увлечение игрой на саксофоне. Источник: У.Дж. Клинтон. Моя жизнь. / Пер. с. англ. - М.: Альпина Бизнес Букс, 2005. - С. 45. Автор: Дмитрий Голдов Вопрос 2: (pic: 20110734.jpg) В 1970-е годы ИХ лицо изменила "птичья клетка". Назовите ИХ максимально точно. Ответ: Хоккейные вратари. Комментарий: Перед вами фотография легендарного голкипера "Detroit Red Wings" Терри Савчука. "Птичья клетка" - первый вариант, конструктивно близкий к современной маске. С тех пор лица хоккейных вратарей изменились и в игре, и вне игры. Источник: 1. http://hockeyshostka.ucoz.ru/publ/istorija_vratarskoj_maski/4-1-0-17 2. http://www.sports.ru/tribuna/blogs/professional/116787.html Автор: Андрей Солдатов Вопрос 3: Действие рекламного ролика крупной компании происходит летом. В нем обаятельный седовласый бородач делает заказ, после чего поясняет удивленному бармену, что сейчас на каникулах. О рекламе какого товара идет речь? Ответ: "Пепси". Зачет: "Пепси-кола", Pepsi, Pepsi-Cola. Комментарий: Традиционно Санта-Клаус - рекламный символ Coca-Cola. Источник: http://www.thedrum.co.uk/news/2011/07/01/santa-betrays-coke-new-pepsi-advert Автор: Алексей Дуболазов Вопрос 4: <раздатка> bws Caerdydd Иногда эту надпись можно увидеть на втором этаже. Переведите ее на русский язык. Ответ: Кардиффский автобус. Комментарий: Это валлийский язык. Источник: http://www.bus-and-coach-photos.com/picture/number2567.asp Автор: Михаил Савченков Вопрос 5: Восклицательный знак, обозначающий факториал, упрощает жизнь ученым-теоретикам. А вот в ряде эфиопских языков используется перевернутый восклицательный знак, который бы наверняка упростил жизнь известному физику. Кому именно? Ответ: Шелдону Куперу. Комментарий: Перевернутый восклицательный знак обозначает сарказм, которого не понимает персонаж сериала "Теория большого взрыва". Источник: 1. http://ru.wikipedia.org/wiki/Шелдон_Купер 2. http://en.wikipedia.org/wiki/Irony_punctuation Автор: Андрей Солдатов Вопрос 6: <раздатка> 1) "Ведь мне что нужно было? Весело жить и иметь всё, чего душа захочет. А ему? Ему нужна была власть... А мне что? Мне бы только повеселиться. Я ведь, по существу, безобидный человек, попавший под дурное влияние". 2) "Большинство моих коллег... знают, что я представитель сравнительно небольшой страны в Азии. Они также знают, что моя страна в течение многих лет неуклонно проводит политику неприсоединения к блокам и дружбы со всеми государствами, независимо от их идеологии. В своей новой роли я буду по-прежнему стоять на этой объективной позиции и следовать идеалам всеобщей дружбы". Назовите тех, кому принадлежат приведенные высказывания. Ответ: 1. Весельчак У. 2. У Тан. Комментарий: У Тан - Генеральный секретарь ООН, бирманец. Источник: 1. http://www.kirbulychev.ru/alisa/pytechectvie_alicu/pytechectvie_alicu_13.html 2. http://www.un.org/russian/basic/sg/apv1046.pdf Автор: Николай Ужов Вопрос 7: Крайне редкий минерал, основу которого составляет плавленый песок, запрещено собирать. Напишите название этого минерала, содержащее три буквы "Т". Ответ: Тринитит. Комментарий: При первых испытаниях ядерного оружия образовалось небольшое количество характерного минерала, который и сейчас можно найти. Источник: 1. http://en.wikipedia.org/wiki/Trinitite 2. http://xronofag.blogspot.com/2004/11/blog-post_110205258164136270.html Автор: Андрей Солдатов Вопрос 8: <раздатка> Stacanovismo В итальянском языке есть слово "Stacanovismo" [Стакановизмо]. Рассуждая об этом, Дмитрий Верхотуров обращает внимание на несколько парадоксальную распространенность русского фразеологизма. Напишите имя собственное, употребляемое в нем. Ответ: Карло. Источник: 1. http://it.wikipedia.org/wiki/Stacanovismo 2. http://schriftsteller.livejournal.com/551050.html Автор: Михаил Савченков Вопрос 9: Девиз какого штата - "Надежда"? Ответ: Род-Айленд. Комментарий: Этот короткий вопрос поведал о том, что у самого маленького штата самый маленький девиз. Источник: http://en.wikipedia.org/wiki/Rhode_Island Автор: Михаил Савченков Вопрос 10: На картине Александра Дейнеки изображены хвосты самолетов, стоящих в ряд в советском аэропорту. Названием этой картины стала пушкинская строка. Напишите ее. Ответ: "Все флаги в гости будут к нам". Источник: http://www.deineka.ru/work-vse_flagi_v_gosti.php Автор: Андрей Солдатов Вопрос 11: <раздатка> С начальной цифрой я сравню владыку Рима: Все люди без него - пустые, как нули. Перед вами фрагмент произведения Тулсидаса, поэта, родившегося в первой половине шестнадцатого века. Какое слово мы заменили в приведенной цитате? Ответ: Раму. Зачет: Рама. Комментарий: В Европе ноль еще не имел серьезного распространения, логично, что индус пишет о Раме. Источник: 1. Всемирная эпиграмма: антология. - Т. 1. - СПб.: Политехника, 1998. - С. 574. 2. http://ru.wikipedia.org/wiki/Тулсидас Автор: Михаил Савченков Вопрос 12: Известный человек обладал феноменальным слухом. В его творческом наследии есть, в том числе, научные работы 1914-1917 годов, в которых он транскрибировал пение птиц. Назовите этого человека. Ответ: Велимир Хлебников. Зачет: Виктор Хлебников. Комментарий: Считается, что "Пинь-пинь-пинь!", которое тарарахнул зинзивер, - это отголосок занятий орнитологией. Источник: http://magazines.russ.ru/zz/2009/17/il14.html Автор: Андрей Солдатов Вопрос 13: Некоторые врачи полагают, что спорт и спиртное несовместимы. Однако Александр Севидов, тренировавший кишиневский "Зимбру", вспоминает, как однажды зимой вино использовали перед матчем. Для чего? Ответ: Для нанесения разметки на снегу. Зачет: По смыслу. Комментарий: Обычно используют красный мяч, а тут еще и решили разметку сделать красной на белом. Источник: http://football.kulichki.net/worldnews/news.htm?171264 Автор: Михаил Савченков Вопрос 14: Александр Мурылёв неплохо знал несовершенное законодательство, и в первой половине 90-х совершил серию убийств жителей городских низов. Он стал одним из первых ИХ в истории России. Уроженец восточного побережья США Филипп Пэйтон был ИМ в буквальном смысле, и сейчас его уважительно называют отцом... Чего? Ответ: Гарлема. Комментарий: Мурылёв нечестным путем завладел квартирами нескольких несчастных людей, став первым известным "черным риэлтором". А вот Филипп Пэйтон пробился с самых низов честно, завладел несколькими домами на Манхэттене и селил других чернокожих. Сейчас его называют отцом Гарлема. Источник: 1. http://ru.wikipedia.org/wiki/Мурылёв,_Александр_Владимирович 2. http://en.wikipedia.org/wiki/Philip_A._Payton,_Jr. Автор: Андрей Солдатов Вопрос 15: <раздатка> В пучину кидаться за перлом бесценным, К миражу пустыни влачиться в тоске, Затем, чтоб оставить, ушедши со сцены, <...> и <...>. Перед вами фрагмент стихотворения Леопольда Стаффа в переводе Асара Эппеля. Заполните пропуски. Ответ: Круги на воде, следы на песке. Источник: http://magazines.russ.ru/inostran/2005/10/ep12.html Автор: Андрей Солдатов Тур: 10 тур. Команда Елены Ковальчук Дата: 07-Apr-2012 Инфо: Команда выражает благодарность Ольге Берёзкиной за ряд ценных замечаний и особенно Андрею Абрамову за очень большую помощь в работе над пакетом. Вопрос 1: [Нулевой вопрос] Автор вопроса принял предложение своей знакомой, пообещав ей целое ведерко... Чего? Ответ: Слив. Комментарий: Не требуется. Источник: ЛОАВ. Автор: Михаил Болотский Вопрос 2: <раздатка> Вот она, бледная, словно графиня, Движется мелким шажком, То изогнется, то робко застынет, Каждым дрожа завитком. Владимир Гаухман, автор поэмы, отрывок из которой мы вам раздали, не был профессиональным поэтом. Чьи услуги предлагало открытое им в 2000 году частное предприятие? Ответ: Врача-дерматовенеролога. Зачет: По смыслу, с упоминанием венеролога или хотя бы дерматолога. Комментарий: Она - бледная спирохета. Источник: 1. http://www.artkavun.kherson.ua/vladimir-gauhman.htm 2. http://shkolazhizni.ru/blog/200281/ 3. http://blogs.mail.ru/mail/yunata09/F8E18B0C8BB060B.html Автор: Михаил Розенблюм Вопрос 3: (pic: 20110735.jpg) Закрытое от вас название книги, которую читает собака на карикатуре Сергея Корсуна, состоит из двух слов разной длины, начинающихся на одну букву. Воспроизведите это название. Ответ: "Корейская кухня". Комментарий: Стивен Кинг - признанный мастер жанра horror. Источник: http://caricatura.ru/top/best/url/black/korsun/206/ Автор: Леонид Эдлин Вопрос 4: (pic: 20110736.jpg) Начнем стандартно: перед вами раздатка. Каждый из прямоугольников закрывает больше, чем ничего, но мы всё же просим вас проанализировать картинку и воспроизвести то, что на ней зашифровано. Ответ: "Для любого ε>0 существует δ>0". Зачет: С кванторами в виде значков и даже с пропуском ">0". Комментарий: Это стандартный зачин теорем из математического анализа. Квантор всеобщности - перевернутое латинское A (первая латинская буква), эпсилон - пятая греческая буква, квантор существования - отраженная латинская E (пятая латинская буква), дельта - четвертая греческая буква. Источник: ЛОАВ. Автор: Михаил Розенблюм Вопрос 5: В афоризме о политике Пьера Буаста говорится о насилии и хитрости, а в произведении 1911 года фигурирует предмет одежды. Назовите любую из тех, что не названы нами, но упомянуты в обоих источниках. Ответ: Правая рука, левая рука. Комментарий: "Если насилие - это правая рука политики, то хитрость - ее левая рука" (Пьер Буаст). "Я на правую руку надела // Перчатку с левой руки" (Анна Ахматова). Источник: 1. http://www.foxdesign.ru/aphorism/topic/t_policy3.html 2. http://www.akhmatova.org/verses/verses/568.htm Автор: Леонид Эдлин, в редакции Михаила Розенблюма Вопрос 6: Некоторое время назад автор вопроса в разгар попойки обратил внимание соседа по столу на то, что служба военной разведки одной из стран называется довольно странно, особенно если вспомнить имя известного предателя военных секретов той же страны. Впрочем, собеседника, выпившего уже немало, этот парадокс не смутил. Как зовут троюродную бабушку автора вопроса? Ответ: Эсфирь. Зачет: Эстер. Комментарий: Служба военной разведки Израиля зовется АМАН, а упомянутый предатель - Мордехай Вануну. Дело было во время праздника Пурим, правило которого гласит: напиться так, чтобы не отличать гонителя евреев Амана от их защитника Мордехая. Источник: 1. http://www.agentura.ru/dossier/izrail/aman/ 2. http://www.rg.ru/2004/01/22/vanunu.html 3. http://holidays.netzah.org/purim.php 4. ЛОАВ. Автор: Михаил Розенблюм Вопрос 7: (pic: 20110737.jpg) Перед вами серия мягких игрушек. Внутри каждой находится мешочек с наполнителем, благодаря чему игрушка может выполнять дополнительную функцию. Как называется эта серия? Ответ: "Хот дог". Комментарий: Разогревая наполнитель в микроволновой печи, можно превратить игрушку в грелку. Источник: 1. http://images.yandex.ru/yandsearch?text=грелки+хот+дог 2. http://pichshop.ru/catalog/uyutnyie_shtuki/grelka_hot_dog_labrador/ Автор: Елена Чухраёва Вопрос 8: Надпись на дверях церкви "При входе в храм отключите свои телефоны" показалась литератору Льву Рубинштейну стилистически неточной. Он отмечает, что в ее тексте следовало бы сделать некоторое изменение, никак не меняющее суммарный смысл сказанного. Что это за изменение? Ответ: Следовало написать "телефоны свои". Комментарий: От перемены мест слагаемых сумма не меняется. Рубинштейн пишет: "Что-то в этой словесной конструкции меня царапнуло - порядок слов. В этом контексте, разумеется, следовало бы написать "отключите телефоны свои"". Источник: http://www.grani.ru/Culture/essay/m.112563.html Автор: Михаил Розенблюм Вопрос 9: Внимание, в вопросе есть замены. Несколько лет назад одна международная спортивная федерация СДЕЛАЛА ЭТО с младшими АЛЬФАМИ. С 1976 года известно, что СДЕЛАТЬ ЭТО можно с любой АЛЬФОЙ. Что мы заменили словосочетанием "СДЕЛАТЬ ЭТО"? Ответ: Раскрасить в четыре цвета. Комментарий: АЛЬФА/АЛЬФЫ - карта/карты. Федерация - Всемирная Федерация Бриджа, раскрасившая на официальной колоде младшие карты каждой из мастей в свой цвет. Гипотеза четырех красок была доказана в 1976 году Аппелем и Хакеном. Источник: 1. ЛОАВ. 2. http://www.krugosvet.ru/node/39266?page=0,2 Автор: Михаил Розенблюм Вопрос 10: <раздатка> Век шествует путем своим... Цитату из Баратынского "Век шествует путем своим" автор статьи, опубликованной в январе 2009 года на одном из спортивных сайтов, использовал для заголовка. Что он добавил к тексту, который вы видите на раздатке? Ответ: Букву "й". Комментарий: В тексте - Вейк. Шахматный турнир в Вейк-ан-Зее ежегодно проводится в январе. Источник: http://www.chesspro.ru/_events/2009/weik2.html Автор: Михаил Розенблюм Вопрос 11: Из воспоминаний известного актера. Однажды, вопреки ожиданию, его не встретили на вокзале. Позже выяснилось, что адресатом была получена телеграмма, содержавшая дату приезда и просьбу одуматься. Назовите автора воспоминаний. Ответ: Регимантас Адомайтис. Комментарий: Телеграмма гласила: "Приезжаю двадцать шестого тчк одумайтесь". Источник: http://m-yu-sokolov.livejournal.com/2082311.html Автор: Михаил Розенблюм Вопрос 12: Прочитав издалека вывеску некоего отечественного заведения, автор вопроса предположил, что оно специализируется на банковских услугах, а подойдя поближе, увидел подзаголовок-расшифровку из двух слов и понял, что на самом деле это точка общепита. Какое основное блюдо там подавали? Ответ: Блины. Комментарий: (pic: 20110738.jpg) Огромная надпись на вывеске гласила "Рубли", а мелкий подзаголовок - "Русские блины". Источник: 1. ЛОАВ. 2. http://fotoshanson.ru/userphotos/fotoprikoly-3/747 Автор: Дмитрий Панин Вопрос 13: Согласно шутке интернет-пользователя, недавно производители дополнительного оборудования для автомобилей выпустили ЭТО под названием "Дмитрий Медведев" с увеличенной гарантией - четыре года. Что ЭТО? Ответ: Подогрев сидений. Комментарий: Четыре года грел кресло для Путина. Источник: http://vk.com/wall-29428419_206770 Автор: Андрей Колчевников Вопрос 14: Внимание, в вопросе есть замена. Лион Фейхтвангер был известен своим интересом к России. Так, в его романе "Успех" фигурирует ленивец Орлов, произведший сильное впечатление на баварского министра. Что заменил автор романа? Ответ: Фамилию "Потемкин". Комментарий: Судя по всему, Фейхтвангер неплохо знал российскую историю. А автор вопроса заменил броненосца на ленивца. Фамилия министра, о котором идет речь, - Кленк. Вопрос посвящается капитану команды "Gambler". Источник: http://www.jewish-library.ru/feyhtvanger/uspeh/4/1.htm Автор: Михаил Розенблюм Вопрос 15: (pic: 20110739.jpg) [Ведущему: а) предупредить, что раздатка обрезана для экономии бумаги, и к содержанию вопроса это отношения не имеет; б) наличие кавычек никак при чтении не проявлять.] Дуплет. 1. Внимание, в вопросе есть замена. Перед вами АЛЬФА "АЛЬФА". Назовите АЛЬФУ двумя словами. 2. Мы надеемся, что хотя бы ОНИ уже располагают правильным ответом на первый вопрос. Какие два слова мы заменили на "ОНИ"? Ответ: 1. Открытая книга. 2. Два капитана. Комментарий: Романы Вениамина Каверина. Источник: http://nnm.ru/blogs/typ84/knigi-veniamin-kaverin-sobranie-sochineniy-1920-1989/ Автор: Михаил Розенблюм Вопрос 16: Внимание, приуменьшенная цитата! "Лучше уж делай что-нибудь другое, можешь, например, из мести высечь море розгами". Назовите произведение, из которого мы взяли оригинал этой цитаты. Ответ: "Солярис". Комментарий: В исходной цитате вместо "море" стоит "океан". Источник: http://www.phantastike.ru/books/fiction/solyaris.zip Автор: Михаил Болотский Тур: 11 тур. Команда Бориса Окуня Дата: 07-Apr-2012 Вопрос 1: [Во время раздачи играет мелодия.] <раздатка> Nyihahák Вам на столы раздали венгерский вариант ИХ названия. На многих других языках ИХ названия очень похожи между собой. Назовите ИХ по-русски. Ответ: Гуигнгнмы. Комментарий: По распространенному мнению, словом "гуигнгнмы" Свифт воспроизводит лошадиное ржание. Прослушанная вами мелодия называется "Wild Horse", да и в начале ее можно услышать ржание. Источник: 1. http://ru.wikipedia.org/wiki/Гуигнгнмы 2. http://hu.wikipedia.org/wiki/Nyihah%C3%A1k Автор: Владимир Ксенофонтов Вопрос 2: Джеймс Джойс так описывает священника: "Казалось, он пользовался хитростью, изворотливостью только [ПРОПУСК], без радости и без ненависти, не думая о том, что в них дурного...". Какие четыре слова мы пропустили? Ответ: "... для вящей славы Божией...". Зачет: "... к вящей славе Господней..." и т.д. с точностью до перевода. Комментарий: Хитрый и изворотливый священник - конечно, иезуит. "Для вящей славы Божией" - девиз этого ордена. Источник: Дж. Джойс. Портрет Художника в юности. Автор: Юрий Кузьменко Вопрос 3: Зорге в Шанхае Поразило мужество Местных жителей, Не испугавшихся ИХ. О чем же он так писал? Ответ: [Японские] танки. Комментарий: И вопрос написан в размере танки. Автор: Виктор Мялов (Днепропетровск) Вопрос 4: (pic: 20110740.jpg) Перед вами изображение епископской кафедры, приписываемой скульптору Николаусу Герхарту. Ее украшают статуи четырех учителей церкви: Августина Блаженного, Амвросия Медиоланского, Иеронима Стридонского и Григория Великого. Считается, что при работе над кафедрой Герхарт СДЕЛАЛ ЭТО. Скажите, кто СДЕЛАЛ ЭТО, согласно произведению XIX века. Ответ: Унтер-офицерская вдова. Комментарий: Скульптор, предположительно, высек сам себя выглядывающим справа снизу. Источник: 1. http://drakosh-a.livejournal.com/74743.html 2. Н.В. Гоголь. Ревизор. Автор: Владимир Ксенофонтов Вопрос 5: Внимание, цитата из рассказа Бунина: "Он вдруг ожил и бесовски разыгрался: <...> тяжелый, как булыжник, набитый мной до круглоты, до отказа, <...> он, в диком и резком веселье, то мчался на меня, на койку и бил лбом в ножку койки, то, подпрыгнув, кубарем летел под умывальник, а оттуда к двери, а от двери под иллюминатор...". Какие два слова произносит герой в конце этого рассказа? Ответ: "Ну, погоди". Комментарий: Описание битвы с чемоданом в каюте парохода очень напоминает подобную сцену в мультфильме "Ну, погоди!", оканчивающуюся этим восклицанием волка. Источник: 1. И.А. Бунин. История с чемоданом. 2. "Ну, погоди!", выпуск 7. Автор: Софья Филина Вопрос 6: Персонаж Игоря Курая, живущий в чужой стране, удивился тому, что люди там устанавливают крепкие металлические двери с простыми, ненадежными замками. Однако сосед рассказал ему, что такие двери ставятся не от грабителей, а на случай... Чего? Ответ: Землетрясения. Комментарий: При землетрясении металлическая дверь в металлическом косяке не заклинит. Источник: http://www.litsovet.ru/index.php/material.read?material_id=348870 Автор: Илья Шапиро (Иерусалим) Вопрос 7: (pic: 20110741.jpg) Неподалеку от этого места находится объект протяженностью около семи километров. В какой европейской стране сделана эта фотография? Ответ: Бельгия. Комментарий: На фото вода характерного цвета у берегов реки О'Руж, давшей название известному повороту на трассе Спа-Франкоршам в Бельгии. Источник: http://en.wikipedia.org/wiki/Circuit_de_Spa-Francorchamps Автор: Владимир Ксенофонтов Вопрос 8: [Ведущему: очень четко прочитать "Queens".] Первоначально он предполагал завести 12 детей: 6 родных и 6 приемных. Это помогло бы проверить его гипотезу. А несколько лет назад он написал книгу "Queens". Назовите его фамилию. Ответ: Полгар. Комментарий: Ласло Полгар полагал, что наследственных факторов, связанных с умственными способностями, нет. В момент рождения всякое нормальное дитя располагает огромным творческим потенциалом, который остается нереализованным из-за примитивной системы дошкольного и школьного воспитания и образования. Провести эксперимент на 12 детях не получилось, но всему миру стали известны его три дочери. Источник: 1. http://www.chesslibrary.ru/publ/ehnciklopedija_shakhmat/boris_spasskij/glava_2_matchi_turniry_soperniki_chast_2/14-1-0-99 2. http://ru.wikipedia.org/wiki/Полгар,_Ласло_(педагог) Автор: Владимир Ксенофонтов Вопрос 9: В воспоминаниях Романа КармЕна немецкий генерал в 1943 году шел по замерзшей АЛЬФЕ с намерением сдаться в плен. В советское время АЛЬФУ называли Пионеркой. Какое слово мы заменили на "АЛЬФА"? Ответ: Царица. Комментарий: Старое название Сталинграда - Царицын, а рядом протекала река Царица. В советское время она называлась Пионерка. Источник: 1. http://ru.wikipedia.org/wiki/Царица_(река) 2. http://militera.lib.ru/memo/russian/karmen_rl/08.html 3. http://ru.wikipedia.org/wiki/6-я_армия_(Германия) Автор: Виктор Мялов (Днепропетровск) Вопрос 10: (pic: 20110742.jpg) Перед вами монета королевства Торгу, образованного на территории Эстонии в 1992 году. Хотя оно просуществовало только до 1993 года, его валюта до сих пор в ходу у местных жителей и ее курс за прошедшие годы вырос в несколько раз, а сейчас он составляет примерно в 5-6 евро. Назовите номинал этой монеты. Ответ: Пол-литра. Зачет: По смыслу. Комментарий: Дальновидный король Кирилл I жестко привязал курс монеты к цене полулитра водки. Ей и сейчас расплачиваются в эстонских барах. Источник: 1. http://dneprovskij.livejournal.com/107864.html 2. http://www.unusualcoins.ee/Moneti%20Estonii/noname61.htm 3. http://www.sota.ee/forum/index.php?topic=895.0 4. http://www.mke.ee/potrebitel/3922-ekspertiza-kakaja-nedorogaja-estonskaja-vodka-samaja-kachestvennaja Автор: Борис Окунь Вопрос 11: По мнению Николая Хренова, он воплощает в себе изъяны советской психологии, включающей потребность вторгаться в чужой приватный мир, производить произвольный, официально не санкционированный контроль за частными лицами, третировать всех волокитой и формализмом. А ЭТО в его руках смотрится угрожающе, как у гробовщика... Назовите ЭТО коротким словом. Ответ: Метр. Комментарий: Он - это почтальон Печкин, который измерял дядю Федора. Источник: http://ru.wikipedia.org/wiki/Почтальон_Печкин Автор: Евгений Кравченко Вопрос 12: В учебнике по высшей математике автор цитирует Мартина Лютера. А какое утверждение упоминается перед этой цитатой? Ответ: Аксиома выбора. Зачет: Теорема Цермело, принцип максимума Хаусдорфа, лемма Цорна. Комментарий: Цитируется фраза "Спасение только верою возможно". Аксиома выбора вызывает неприятие многих математиков. Источник: 1. А.Я. Хелемский. Лекции по функциональному анализу. 2. http://ru.wikipedia.org/wiki/Аксиома_выбора 3. http://ru.wikipedia.org/wiki/Лютер,_Мартин Автор: Юрий Кузьменко Вопрос 13: Американский биографический фильм 1960 года с Курдом Юргенсом в главной роли называется "Моя цель - звезды". Сатирик Заль продолжил название: "... но иногда я ДЕЛАЮ ЭТО". Заметка на сайте Взгляд.ру утверждает, что "Рубин" СДЕЛАЛ ЭТО по итогам жеребьевки. Что мы заменили на "СДЕЛАЛ ЭТО"? Ответ: Попал в Лондон. Комментарий: Фильм посвящен Вернеру фон Брауну, творения которого, действительно, иногда попадали в Лондон. "Рубин" в Лиге Европы вышел на лондонский "Тоттенхэм". Источник: 1. http://www.imdb.com/title/tt0053440/ 2. http://www.lib-mobile.com/br/?b=117319&p=37 3. http://www.vz.ru/sport/2011/8/26/517787.html Автор: Павел Анохин Вопрос 14: <раздатка> - Ты выберешься отсюда? - спросил я. - Если... - начал он, но сразу же умолк. Перед вами немного измененный диалог между двумя сокамерниками из романа Умберто Эко "Маятник Фуко". После этого рассказчик упоминает ложку. Восстановите измененное слово абсолютно точно. Ответ: If. Комментарий: Это отсылка к "Графу Монте-Кристо". Источник: Умберто Эко. Маятник Фуко. Автор: Борис Окунь Вопрос 15: [Ведущему: слегка выделить голосом слово "подъем".] В недавнем фильме, чтобы показать подъем героини по карьерной лестнице, используют ИХ. Назовите ИХ. Ответ: Титры. Комментарий: В фильме "Артист" имя героини указывается всё выше в титрах различных фильмов. Источник: Фильм "Артист". Автор: Борис Окунь Тур: 12 тур. Команда Эльмира Валеева Дата: 07-Apr-2012 Инфо: Команда благодарит Евгения Поникарова, Александра Чижова, Андрея Супрановича и Мустафу Умерова за ценные советы и замечания при подготовке этого пакета. Вопрос 1: [Нулевой вопрос] (pic: 20110743.jpg) Какое слово мы закрыли на картинке? Ответ: Чумовые. Комментарий: (pic: 20110744.jpg) Источник: http://www.shoki.ru/?p=4989 Автор: Галина Лазарева Вопрос 2: Продолжим тему. Героиня рассказа Анны Лист утверждает, что из рабочих графиков получился "чумовой шалаш". Можно предположить, что героиня при этом путает шалаш с чумом. Аналогичную ошибку совершил однажды и другой известный герой. Мы не спрашиваем вас, что мы заменили словом "шалаш". Ответьте, что мы заменили словом "чум". Ответ: Типи. Комментарий: Словом "шалаш" мы заменили слово "вигвам". Героиня Анны Лист, говоря о том, как выглядят графики, упоминает "типичный вигвам", но, как и Шарик из "Простоквашино", ошибается - судя по описанию, это не вигвам, а другая "индейская национальная народная изба" - типи. Источник: 1. http://www.proza.ru/2010/07/14/1074 2. http://ru.wikipedia.org/wiki/Типи Автор: Эльмир Валеев Вопрос 3: На рекламной листовке одного из российских банков бросается в глаза надпись: "Своя машина. Кто "за"?" А фотография на этой же листовке запечатлела множество поднятых вверх рук, сжимающих ЭТО. Назовите ЭТО. Ответ: Поручни в автобусе (вагоне метро, троллейбусе). Зачет: По смыслу. Автор: Сергей Евдокимов Вопрос 4: (pic: 20110745.jpg) В июне 2008 года член городского совета Индианаполиса Доррис Минтон-МакНейл была арестована за нападение на полицейского. В связи с этим на одном американском сайте появился шуточный фотоколлаж, из которого следовало, что между АЛЬФОЙ и БЕТОЙ лежат всего две "маргариты". Занятие, которое отличается от БЕТЫ только пробелом, прославило другую Маргариту. Назовите это занятие. Ответ: Бадминтон. Зачет: Badminton. Комментарий: В коллаже говорилось о Good Minton и Bad Minton. Первой чемпионкой СССР по бадминтону была Маргарита Зарубо. Источник: 1. http://www.theindychannel.com/news/16680967/detail.html 2. http://www.bartlies.com/2008/06/23/good-minton-bad-minton/ Автор: Эльмир Валеев Вопрос 5: <раздатка> Такую ткань узорную творя Из воздуха, как патока, густого, [Пропуск] судьбы вплелась в ее основу, И каждое несказанное слово Впаялось в память каплей янтаря. При попытке ввести в поисковую систему Google два пропущенных слова, она предлагает добавить к ним слова "интересное", "новенькое" или "умное". Восстановите пропуск. Ответ: Что нить. Комментарий: Google подразумевал, конечно, распространенную в Интернете версию написания слова "что-нибудь". Источник: 1. http://tzitzitlini.livejournal.com/45114.html 2. Google. Автор: Эльмир Валеев Вопрос 6: Название магазина одежды в Ростовской области почти не отличается от имени известного литературного персонажа. Несмотря на неоднозначность этого названия, один из пользователей Живого Журнала отметил, что со слоганом магазина "Искусство выделяться из толпы" как будто не поспоришь. Восстановите это название. Ответ: Квазимода. Комментарий: Имя "Квазимодо" значит "как будто". Источник: http://www.liveinternet.ru/users/4161105/post161088057/ Автор: Эльмир Валеев Вопрос 7: Внимание, в вопросе есть замена. В августе православные отмечают день чудотворной иконы Панагия Смогла. Название иконы объясняется тем, что, по преданию, она стала причиной основания монашеской обители на Черной горе. Напишите замененное нами слово. Ответ: Сумела. Комментарий: Икона, как и монастырь в Турции, называются Панагия Сумела. "Черная" по-гречески - "мела" (вспомним меланому, Меланезию и т.д.). "Су-мела" переводится как "с черной (горы)". Источник: http://ru.wikipedia.org/wiki/Панагия_Сумела Автор: Галина Лазарева Вопрос 8: Доктор наук Черный, объясняя ИХ феномен, упоминает эксперименты по левитации сверхпроводников в магнитном поле, а также узоры, образуемые металлическими опилками в школьном опыте с магнитом. ИХ нередко сравнивают с ИКСОМ. В 1969 году ОНИ действительно появлялись на ИКСЕ, хотя вряд ли вы ИХ слышали. Назовите ИХ двумя словами. Ответ: Кольца Сатурна. Комментарий: Во втором случае речь о музыкальном альбоме Мишель О'Мали, который назывался "Saturn Rings". ИКС - виниловая пластинка. Источник: 1. http://www.nkj.ru/archive/articles/710/ 2. http://www.allmusic.com/album/saturn-rings-r844133 Автор: Сергей Евдокимов Вопрос 9: Внимание, в вопросе есть замены. В начале XX века некоторые известные люди размещали объявления о том, что для достижения высоких целей им нужны ВОДОЛАЗЫ. В последние годы многие пользователи Интернета также дают объявления о том, что им нужны ВОДОЛАЗЫ для достижения других высоких целей. Какое слово мы заменили на "ВОДОЛАЗЫ"? Ответ: Лайки. Комментарий: В первом случае речь о полярных исследователях, которым нужны были собаки определенных пород, в т.ч. лайки, в качестве ездовых. Цели их при этом были не только морально высоки, но и находились в высоких широтах. Во втором - о "лайках", как производном от "like", специальных кнопках в социальных сетях; таким образом реализуется продвижение человека/продукта в социалках. Источник: Запрос в Google "нужны лайки". Автор: Эльмир Валеев Вопрос 10: Фотогалерея на сайте РИА "Новости", посвященная известной личности, умершей в 2000 году, называется "Человек, который первым попал в космос". В двух словах этого названия мы пропустили по букве. Напишите эти буквы. Ответ: с, е. Комментарий: Галерея называется "Человек, который первым поспал в космосе". Полет Титова, в отличие от полета Гагарина, продолжался больше суток. Источник: http://ria.ru/photolents/20100911/274293514.html Автор: Эльмир Валеев Вопрос 11: Внимание, в вопросе есть замены. На карикатуре в журнале "Нью Йоркер" издатель держит в руках открытую на первой странице рукопись романа и раздраженно говорит: "Вы уж определитесь: либо так, либо эдак. Не бывает, чтобы и то и другое одновременно". Догадавшись, что мы заменили на "так" и "эдак", назовите писателя. Ответ: Чарльз Диккенс. Комментарий: Подразумеваются первые фразы "Повести о двух городах": "Это было самое прекрасное время, это было самое злосчастное время...". Источник: http://www.newyorker.com/online/blogs/cartoonists/2012/02/a-far-far-better-cartoon-gag.html Автор: Галина Лазарева Вопрос 12: Основой рациона этого животного, как ни странно, служат насекомые, ящерицы, змеи и другая мелкая живность. А его название всего одной буквой отличает его от весьма популярного в нашей стране персонажа. Назовите это животное. Ответ: Медоед. Комментарий: Медоед - хищник из семейства куньих, мед ест не так уж часто. Популярный персонаж - медвед. Источник: http://ru.wikipedia.org/wiki/Медоед Автор: Сергей Евдокимов Вопрос 13: В вопросе есть замены. Прослушайте стихотворение Овсея Дриза: Слегка блеснет, и серое опять. Мои глаза слабеют постепенно, Но не могу, не в силах перестать ДЕЛАТЬ ЭТО. Герой современной сказки, попавший в мир, населенный фольклорными персонажами, ДЕЛАЕТ ЭТО, применяя знание физики, чтобы одержать окончательную победу над врагом. Какие пять слов, образующих идиому, мы заменили на "ДЕЛАТЬ ЭТО"? Ответ: Искать иголку в стоге сена. Комментарий: Герой фильма "Там, на неведомых дорожках", снятой по сказке Успенского "Вниз по волшебной реке", искал иголку в стоге сена при помощи ножниц, т.к. они намагничены. Источник: 1. О. Дриз. Белое пламя: Стихи. - М.: Советский писатель, 1990. - С. 3. 2. http://www.youtube.com/watch?v=Vi2bBaf-i4M Автор: Инна Грановская Вопрос 14: В начале широко цитируемого, но малоизвестного стихотворения гроза косым стремительным углом падает на землю. А заканчивается стихотворение такими строками: Устало высохла трава. И снова тишь. И снова мир. Как равнодушье, как [ПРОПУСК]. Заполните пропуск. Ответ: Овал. Комментарий: Две последние строки этого стихотворения Когана общеизвестны: "Я с детства не любил овал! / Я с детства угол рисовал!". Источник: http://www.stihi.ru/2011/05/10/2675 Автор: Галина Лазарева Вопрос 15: В вопросе есть замены. ПЕРВЫЕ ИКСЫ известны давно и упоминаются, в том числе, в Ветхом Завете. ВТОРЫЕ ИКСЫ впервые были изготовлены, если верить Википедии, в начале XIX века, однако широкое распространение получили несколько позже. Для изготовления ВТОРЫХ ПЕРВЫХ ИКСОВ один сайт предлагает использовать компас, картон, ножницы, клей и кусочек проволоки. Что мы заменили на "ВТОРЫЕ ПЕРВЫЕ ИКСЫ"? Ответ: Наручные солнечные часы. Источник: 1. http://ru.wikipedia.org/wiki/Солнечные_часы 2. http://ru.wikipedia.org/wiki/Наручные_часы 3. http://uchifiziku.ru/2012/03/23/naruchnye-solnechnye-chasy/ Автор: Сергей Евдокимов Вопрос 16: Искусство инэмури облегчает японцам жизнь в условиях постоянного стресса. В одном известном отечественном произведении присутствует персонаж Ерёма, который, судя по всему, владеет искусством инэмури. В этом вопросе мы заменили одну букву. Восстановите измененное слово в изначальном виде. Ответ: Дрёма. Комментарий: Инэмури - это искусство спать в любое время и любом месте, в том числе сидя и стоя. Персонажа уже упоминавшегося сегодня фильма "Там, на неведомых дорожках", который постоянно спит, вполне логично зовут Дрёма. Источник: 1. http://ru.wikipedia.org/wiki/Инэмури 2. http://www.youtube.com/watch?v=lXtbcvqky6M Автор: Галина Лазарева, Эльмир Валеев Тур: 13 тур. Команда Егора Игнатенкова Дата: 16-Jun-2012 Инфо: Мы благодарим за тестирование наших вопросов Сергея Ефимова, Григория Алхазова, Евгения Миротина, Константина Науменко, Константина Кнопа. Вопрос 1: В одном из заданий викторины на сайте gogol200.ru предлагались три варианта ответа: "Шинель", "Тараса Бульбу" и "второй том "Мертвых душ"". Сам вопрос состоял из трех слов, первое из которых - Гоголь. Восстановите остальные два. Ответ: Не сжег. Комментарий: Тарас Бульба был без кавычек. Его он тоже сжег - в конце произведения. Источник: Личные воспоминания автора о сайте http://www.gogol200.ru. Автор: Илья Богданов Вопрос 2: Первые модели вертолета, получившего неофициальное название ИКС, не очень хорошо показали себя в горных районах. По одной из версий, из-за этого появился известный анекдот. Что мы заменили на "ИКС"? Ответ: Крокодил. Комментарий: Вертолет - Ми-24. Вероятно, именно из-за этого вертолета появился анекдот о том, что крокодилы летают, только низЭнько-низЭнько. Источник: http://www.wartechnic.ru/countries/russia/aviation/mil/mi24/description Автор: Илья Богданов Вопрос 3: Можно сказать, что в иудаизме при принесении жертвы всесожжения жертвователь должен был отбросить корыта. Какие три слова мы заменили на два в предыдущем предложении? Ответ: Отдать Богу тушу. Комментарий: При принесении жертвы всесожжения животное сжигалось целиком, жертвователь ничего не оставлял себе, всё принадлежало Богу, в том числе и туша животного. Источник: http://ru.wikipedia.org/wiki/Жертвоприношения_в_иудаизме Автор: Егор Игнатенков Вопрос 4: В отеле, где должна была проживать сборная Чехии на чемпионате Европы по футболу, обнаружили болезнетворные бактерии. Корреспондент канала "Спорт 1" сказал, что чешской сборной действительно следует их опасаться, поскольку некий показатель у нее равен 15. Назовите этот показатель. Ответ: Количество легионеров в заявке. Зачет: По слову "легионер" без неверных уточнений. Комментарий: В отеле были обнаружены бактерии legionella, от которых можно заразиться так называемой болезнью легионеров. Источник: Сюжет в новостной программе канала "Спорт 1". Автор: Егор Игнатенков Вопрос 5: <раздатка> В одном из эпизодов книги Лазарчука, действие которой происходит в 1945 году, герои опасаются, что железная дорога может быть взорвана германскими партизанами. Какое слово мы заменили в этом вопросе? Ответьте на розданный вопрос. Ответ: Титовскими. Комментарий: То есть партизанами Иосипа Броз Тито. Дело происходит в Хорватии. Герман Титов - космонавт. Источник: А. Лазарчук. "Штурмфогель". Глава "Хорватия. 8 марта 1945". Автор: Илья Богданов, Константин Кноп Вопрос 6: (pic: 20110746.jpg) Перед вами американская карикатура. Подпись под ней состоит из двух слов. Напишите их. Ответ: "Tsar wars". Источник: (pic: 20110747.jpg) Автор: Михаил Калашников Вопрос 7: Рассказывая о кризисе в Аргентине, автор статьи в журнале "Вокруг света" вспоминает, как иной раз слышал в центре города звук ЕГО. На сайте ebay.com на момент написания вопроса самый дорогой ОН мог бы обойтись в 55 долларов. Назовите ЕГО двумя словами. Ответ: Аукционный молоток. Зачет: Молоток аукциониста. Комментарий: На интернет-аукционе ebay можно купить и антикварные аукционные молотки. Источник: 1. http://www.vokrugsveta.ru/vs/article/6540/ 2. http://www.ebay.com/itm/VINTAGE-AUCTION-JUDGES-COURT-WOODEN-HAMMER-GAVEL-/220711806026 Автор: Михаил Калашников Вопрос 8: Наталья Трауберг, говоря об анемонах, упоминает то, что получается на стыке, если ЕЕ свернуть в трубку. За последние 250 лет тройную ЕЕ наблюдали меньше десятка ученых. Назовите ЕЕ. Ответ: Радуга. Комментарий: Цвет анемонов - как раз фиолетово-красный. Источник: 1. Наталья Трауберг. Сама жизнь. 2. http://www.physorg.com/news/2011-10-myth-reality-photos-triple-rainbows.html Автор: Михаил Калашников Вопрос 9: В вопросе слова "ИКС", "ГЛАЗ" и "СЛИПАЕТСЯ" - замены. По мнению автора вопроса, ИКС - это ГЛАЗ, который действительно СЛИПАЕТСЯ. ИКС часто использовался в бухгалтерии, а с 1583 года использование ИКСА было запрещено. Назовите ИКС несклоняемым словом. Ответ: Кипу. Комментарий: Про кипу действительно можно сказать, что этот язык заплетается. Источник: 1. http://en.wikipedia.org/wiki/Quipu 2. http://www.kuprienko.info/fernando-murillo-de-la-cerda-carta-sobre-los-caracteres-usados-por-los-indios-antes-de-la-conquista-1589/ Автор: Михаил Калашников Вопрос 10: Валентина Капуано едва не погибла из-за того, что один человек решил подобраться поближе к дому своего друга и поприветствовать его гудком сирены. Ответьте, в каком году у Валентины Капуано умер двоюродный дедушка. Ответ: 1912. Комментарий: Капитан Скеттино подобрался слишком близко к берегу и посадил лайнер "Коста Конкордиа" на мель. Джованни Капуано плыл на "Титанике", тоже пострадавшем от самоуверенности капитана. Источник: 1. http://www.telegraph.co.uk/news/worldnews/europe/italy/9016769/Cruise-disaster-Captain-neared-Italian-rocks-to-greet-friend-on-shore.html 2. http://lenta.ru/news/2012/01/18/titanic/ Автор: Михаил Калашников Вопрос 11: <раздатка> Trimmatom nanus На одном сайте утверждается, что самая короткая ОНА - Trimmatom nanus. Другая самая короткая ОНА состоит из десяти... Чего именно? Ответ: Костей домино. Комментарий: ОНА - рыба. Источник: 1. http://chemworld.narod.ru/museum/fish.html 2. Личные вычисления автора. Автор: Илья Богданов Вопрос 12: Сварливый герой англоязычного сериала, проведя чудесные рождественские каникулы со своей девушкой, возвращается совершенно преобразившимся. Назовите имя его девушки. Ответ: Кэрол. Зачет: Carol, Caroline, Кэролайн. Комментарий: Прослеживается аллюзия на "Рождественскую песнь" Диккенса, в оригинале - "Christmas carol". Источник: Т/с "Office US", s03, ep10/11. Автор: Иделия Айзятулова Вопрос 13: Журналист, пишущий о Кубке Америки, отмечает, что после ЭТОГО не может быть никаких ничьих. Назовите ЭТО. Ответ: Экватор. Комментарий: Турнир перевалил экватор с началом плей-офф, когда никаких ничьих уже не бывает. Источник: http://www.championat.com/football/article-90496.html Автор: Михаил Калашников Вопрос 14: В своей статье "Столкновение бородатых" Михаил Идов рассказывает, как в 2009 году на одной из улиц Бруклина уничтожили велосипедную дорожку. В подзаголовке статьи фигурируют две категории людей, начинающиеся на одну и ту же букву. Назовите обе категории. Ответ: Хипстеры, хасиды. Комментарий: Хасиды были недовольны засильем хипстеров в районе и приняли меры. Источник: http://www.nymag.com/realestate/neighborhoods/2010/65356/ Автор: Михаил Калашников Вопрос 15: В 1960 году у этой кафедры МГУ было 16 выпускников, а 50 лет спустя - лишь четверо. Статья в журнале "Scientist" рассказывает, что число ученых, занимающихся этой областью науки, составляет сейчас всего несколько сотен, им почти не достается грантов, и количество мест для них в университетах только сокращается. Назовите специализацию этих ученых. Ответ: Палеонтология. Комментарий: Статья так и называется: "Вымирают ли палеонтологи?". Источник: 1. http://www.geol.msu.ru/deps/paleont/vipusk.html 2. http://classic.the-scientist.com/news/display/55888/ Автор: Михаил Калашников Тур: 14 тур. Команда Юлии Силинг Дата: 16-Jun-2012 Инфо: Благодарим за тестирование Александра Бердичевского, Дарью Бронникову и Татьяну Руссит. Вопрос 1: Недавно, когда автор вопроса летел в Лондон, он наблюдал в самолете следующую сцену: стюардесса, проходящая с тележкой мимо одного из пассажиров, в ответ на его просьбу повернулась к своей коллеге, стоявшей в другом конце салона, и дотронулась пальцами до своих глаз. Та кивнула и принесла необходимое. Что именно? Ответ: Лед. Комментарий: Стюардесса предлагала пассажирам напитки. Слова "ice" (лед) и "eyes" (глаза) звучат по-английски почти одинаково. Источник: Личный опыт автора. Автор: Борис Иомдин Вопрос 2: Для того чтобы доказать, что огонь, горевший около миллиона лет назад в пещере Вондерверк, развели древние люди, археологам пришлось доказать, что это не было самовозгорание. Что же могло загореться в глубокой сырой карстовой пещере? Ответ: Помет летучих мышей. Источник: http://elementy.ru/news/431835 Автор: Светлана Бурлак Вопрос 3: В городе Остин, штат Техас, каждый год в одно из воскресений февраля на улицы выходят несколько десятков оркестров, рок-групп и просто одиночных музыкантов. Раньше всех, в 7 утра, начинают свои концерты те из них, кто располагается к югу от центра. Несколько позже вступают те, кто стоит в западной части города, потом на северной окраине. Позже всех, около 9 утра, начинают играть музыканты в кварталах, расположенных прямо к северу от городского центра. Кого таким образом приветствуют музыканты? Ответ: Участников городского марафона. Источник: 1. http://www.youraustinmarathon.com/images/stories/2012/course/2012%20bands.pdf 2. Личные впечатления автора вопроса. Автор: Георгий Бронников Вопрос 4: В городе Остин, штат Техас, есть мост Congress bridge. Каждый день в конце лета в вечерних сумерках вокруг этого моста собираются толпы людей. Туда водят экскурсии и устраивают обзорные лодочные поездки под мост, невзирая даже на неприятный запах. О причинах такого ажиотажа говорили и у нас в Москве. Назовите эту причину. Ответ: Массовый вылет летучих мышей. Зачет: По словам "летучие мыши" и без указания на что-либо кроме полета. Комментарий: Это крупнейшая в Северной Америке городская колония летучих мышей. О летучих мышах говорили мы, когда отвечали на вопрос про помет в пещере. Источник: Личные впечатления автора вопроса. Автор: Георгий Бронников Вопрос 5: Описывая сборы в плавание, Иван Федорович Крузенштерн писал: "Масла взял я малое количество, для того что оно между поворотными кругами обыкновенно...". Мы не спрашиваем, что должно было случиться с маслом. Что такое поворотные круги? Ответ: Тропики. Комментарий: Во времена Крузенштерна их так называли. Автор: Сергей Козинцев Вопрос 6: Книгу "Путешествие вокруг света" Иван Федорович Крузенштерн начинает с предуведомления, в котором содержится более десятка пунктов. Чему посвящена большая часть этих пунктов? Отметим, что если бы книга писалась в наше время, необходимости приводить такое количество пунктов не было бы. Ответ: Единицам измерения. Источник: Упомянутая в вопросе книга. Автор: Сергей Козинцев Вопрос 7: В 2010 году вышла книга, название которой буквально можно перевести так: "Синтаксис устной речи в не-шестиугольных вариантах [ПРОПУСК]...". Заполните пропуск двумя словами. Ответ: "... французского языка". Комментарий: Оригинальное название - "La syntaxe de l'oral dans les variétés non-hexagonales du français". Контур Франции напоминает правильный шестиугольник, поэтому он является символом страны. Французский язык распространен и в других странах. Источник: Drescher, Martina / Neumann-Holzschuh, Ingrid (Hrsg.): La syntaxe de l'oral dans les variétés non-hexagonales du français. 201 S. - Tübingen: Stauffenburg, 2010. Автор: Борис Иомдин Вопрос 8: (pic: 20110748.jpg) Один пользователь Живого Журнала опубликовал вот такую картинку. Восстановите то, что мы от вас закрыли. Ответ: Malevich. Зачет: Malewicz. Источник: http://kurko-vain.livejournal.com/17861.html Автор: Мария Рубинштейн Вопрос 9: (pic: 20110749.jpg) Посмотрите на розданное вам изображение. Чтобы увеличить продажи в дневное время, руководство сеульского супермаркета "Emart" установило перед входом вот такое сооружение. Цель была достигнута, покупателей немедленно стало гораздо больше. И хотя тысячи людей тут же бросились фотографировать это сооружение, привлекло их не оно, а объявленные скидки на товары. С помощью чего можно было получить эти скидки? Ответ: С помощью QR-кода. Зачет: По наличию любого из словосочетаний: QR-код, матричный код, quick response, двумерный штрих-код. Комментарий: Эти выступы вместе с тенями от них ровно в полдень образовывали QR-код, который можно было сфотографировать на смартфон и так получить скидку. Источник: http://www.qrcartist.com/2012/emart-sunny-sale-campaign-shadow-qr-code/ Автор: Мария Рубинштейн Вопрос 10: В этом вопросе картинок-раздаток не будет. "Соединяем с любимым", "Улучшаем вид из окна", "Добавляем героизма". Это далеко не исчерпывающий список услуг, на сайте этого учреждения есть и другие, более привычные. С чем же именно там так работают? Ответ: С фотографиями. Источник: http://www.netprint.ru/ru/478 Автор: Мария Рубинштейн Вопрос 11: Вопрос американского профессора, обращенный к российским коллегам: "Почему они у вас садятся, а потом стоят?". Мы не просим ответить на сам вопрос. Какое слово мы заменили на "они"? Ответ: Самолеты. Комментарий: Лингвист Барбара Парти удивлялась тому, как в русском языке описывается поведение самолетов. Источник: Воспоминания автора вопроса от курса семантики в РГГУ в 2003 г. Автор: Георгий Бронников Вопрос 12: Холли Хантер, Эдриэн Броуди, Изабель Юппер. Назовите город и инструмент, которые объединяют этих людей. Ответ: Канны, пианино. Зачет: Канны, фортепиано. Комментарий: На Каннском кинофестивале Холли Хантер получила приз как лучшая актриса за главную роль в фильме "Пианино", Изабель Юппер - как лучшая актриса за главную роль в фильме "Пианистка", а Эдриэн Броуди сыграл главную роль в фильме "Пианист", получившем Золотую пальмовую ветвь. Источник: http://www.imdb.com Автор: Борис Иомдин Вопрос 13: Мы не знаем, был ли это замысел Георгия Данелии или просто недоразумение, на которое во время съемок не обратили внимание. В любом случае зрители ничего странного в фильме не увидели, для них ситуация выглядела вполне естественно, потому что за время, прошедшее от съемок до премьеры, в Москве успели построить... Что именно? Назовите оба эти объекта в любом порядке. Ответ: "Проспект Вернадского", "Юго-западная". Зачет: Любое понятное указание на нижние станции красной ветки. Комментарий: В финале фильма "Я шагаю по Москве" Володя садился в поезд на "Университете", чтобы ехать во Внуково, но вообще-то на момент съемок "Университет" был конечной станцией. Внимательные зрители даже могут разглядеть там надпись "Посадки нет". А когда фильм вышел в кино, уже были построены две следующие станции. Источник: http://ru.wikipedia.org/wiki/Я_шагаю_по_Москве Автор: Мария Рубинштейн Вопрос 14: (pic: 20110750.jpg) Часть надписи мы вырезали, но вы все-таки назовите страну, отдых в которой попытались прорекламировать авторы этого рисунка. Ответ: Куба. Комментарий: "Подарите себе незабываемый отдых на Кубе". Источник: http://www.kletka.net/5040-podari-sebe-nezabyvaemyy-otdyh-na-kube.html Автор: Мария Рубинштейн Вопрос 15: Пресуппозицией в лингвистике называется необходимый семантический компонент, обеспечивающий наличие смысла в утверждении. По мнению одного российского лингвиста, понятие пресуппозиции невозможно объяснить без обращения к некоторому другому понятию. Вы с ним, скорее всего, хорошо знакомы. Но даже если нет, вам знаком один связанный с ним вопрос. Воспроизведите этот вопрос, и, так и быть, можете сами на него не отвечать. Ответ: "Ты перестала пить коньяк по утрам?". Источник: http://dimkaguarani.livejournal.com/194989.html Автор: Мария Рубинштейн Тур: 15 тур. Команда Дмитрия Еловенко Дата: 16-Jun-2012 Инфо: Мы благодарим за тестирование Александра Коробейникова, Александра Рождествина, Бориса Моносова, Андрея Марьянского, Михаила Барабаша, Людмилу Тюрину, Евгению Байбакову, Игоря Философова, Екатерину Свешникову, Виктора Мосягина, Кирилла Андреева, Дмитрия Козурова, Светлану Орлову, Владислава Дронова, Дарью Костенко, Артема Сорожкина, Дмитрия Бурова, Марию Колосовскую, Татьяну Харитонову и, наверное, еще какое-то количество людей, о которых мы забыли. Вопрос 1: Герман Мелвилл сравнивает спинной плавник финвала в штиль с НИМ. Назовите ЕГО словом греческого происхождения. Ответ: Гномон. Источник: http://lib.ru/INPROZ/MELWILL/mobidik.txt Автор: Дмитрий Великов Вопрос 2: [Ведущему: кавычки не озвучивать.] Можно сказать, что индийские ОНИ - это, например, ракшАсы и пИшачи, а японские ОНИ - например, мОно и аякАси. Литературовед Людмила Сараскина пишет, что индийские "ОНИ" - это "Дом и мир", а японские "ОНИ" - "В стране водяных". Назовите ИХ. Ответ: Бесы. Комментарий: Сараскина проводит аналогии между произведениями Рабиндраната Тагора и Акутагавы Рюноскэ и "Бесами" Достоевского. Источник: 1. Канал "Культура", 09.11.2011 г., 10:05. ACADEMIA. Спецкурс "Достоевский. "Бесы"". Читает Людмила Сараскина. http://www.tvkultura.ru/issue.html?id=114805 2. Л.И. Сараскина. "Бесы": роман-предупреждение. http://lib.rus.ec/b/333246/read 3. http://ru.wikipedia.org/wiki/Ёкай 4. http://bestiarium.aworld.ru/maska/spisok166.html 5. http://ru.wikipedia.org/wiki/Ракшасы 6. http://ru.wikipedia.org/wiki/Пишачи Автор: Андрей Штефан, Елизавета Штефан Вопрос 3: Дуплет. 1. В переводе Гёте, сделанном в конце XIX века, слово "предназначение" употреблено в значении "набросок". В этой фразе мы заменили несколько букв. Восстановите измененное нами слово. 2. В переводе Гёте, сделанном в конце XIX века, слово "соперничать" употреблено в значении "сопереживать". В этой фразе мы заменили несколько букв. Восстановите измененное нами слово. Ответ: 1. Предначертание. 2. Соревновать. Источник: И.В. Гёте. Путешествие в Италию. / Пер. Н.А. Холодковский. - М.: ОГИ, 2012 (в печати). Автор: Дмитрий Великов Вопрос 4: Внимание, в вопросе есть замены. По преданию, некий сундук периодически нес тех, кто нес его. Можно сказать, что его то несли гады, то он гадил сам. Какие слова мы заменили на "гады" и "гадить"? Ответ: Левиты, левитировать. Комментарий: Ковчег завета несли левиты. Левий и Гад - основатели двух из двенадцати колен израилевых. Источник: 1. http://khazarzar.skeptik.net/thalmud/_tb_ru/sotah.htm 2. http://ru.wikipedia.org/wiki/Ковчег_завета Автор: Елизавета Штефан, Андрей Штефан Вопрос 5: В стихотворении Анны Герасимовой ПЕРВЫЙ думает о службе у ВТОРОГО. И ПЕРВЫЙ, и ВТОРОЙ фигурируют в заимствованиях, сделанных нашими соотечественниками в двадцатом веке. Назовите ПЕРВОГО и ВТОРОГО. Ответ: Буратино, Урфин Джюс. Источник: http://www.umka.ru/liter/stishki.html Автор: Дмитрий Великов Вопрос 6: Последняя глава одной из повестей Юрия Германа начинается словами "На ленинградском". Закончите эту фразу двумя существительными. Ответ: "... фронте перемены". Источник: Ю.П. Герман. Студеное море. http://lib.rus.ec/b/375276/read Автор: Дмитрий Великов Вопрос 7: [Ведущему: "адэ" надо читать четко, с ударением на второй слог, при необходимости можно повторить; по буквам произносить не стоит, однако все три звука должны быть произнесены четко.] По одной из версий, это "адэ" появилось в результате уроков, которые в Ташкенте в 1942 году матрос из-под Балаклавы давал бойцу. Употребление этого "адэ" ныне ясно показывает, что использующий его не имеет никакого отношения к "родственнице". Напишите имя "родственницы". Ответ: Одесса. Комментарий: При подготовке к съемкам фильма "Два бойца" Марк Бернес, будучи родом из Нежина, усваивал одесский говор, общаясь в госпитале с одним из украинских моряков. По версии Михаила Левитина, матрос был из-под Балаклавы, так что поставил ему не вполне аутентичный акцент. Русскоязычные одесситы смягчают звук "д" в слове "Одесса", в украинском же варианте названия города нет "аканья" в первом слоге. Источник: 1. Передача "Марку Бернесу посвящается..." (ведущий - Михаил Левитин), канал "Культура", 26.02.2012 г., 17:05. http://www.tvkultura.ru/news.html?id=921930 2. http://www.kinozal.tv/details.php?id=935823 3. ЛОАВ. Автор: Андрей Штефан, Елизавета Штефан Вопрос 8: [Чтецу: "А дэ?" надо читать четко, с ударением на второй слог, с характерным для вопросительных предложений повышением интонации в конце; наверное, имеет смысл заранее потренироваться. По буквам не произносить, про пробел не упоминать.] В вопросе есть замена. "А дэ?" - это ИКС. Как утверждал Владимир Винниченко после переговоров с Временным правительством, там, где начинается ИКС, заканчивается российская демократия. Какие два слова мы заменили на "ИКС"? Ответ: Украинский вопрос. Комментарий: Украинский вопрос "А дэ?" (русск. "А где?") известен, например, по анекдоту с ключевой фразой "А дэ тут останивка?". Вслед за Максимом Поташевым мы решили поддержать и развить некоторые традиции. Источник: 1. http://proekt-wms.narod.ru/states/vinnichenko-vk.htm 2. http://ru.wikiquote.org/wiki/Владимир_Винниченко Автор: Андрей Штефан, Елизавета Штефан Вопрос 9: Герой Генриха Бёлля вспоминает времена после Первой мировой, когда полбуханки хлеба стоили столько же, сколько и пушка, а ПЕРВОЕ - столько же, сколько ВТОРАЯ. Герой одного детского стихотворения собирается нарвать ПЕРВЫХ со ВТОРОЙ. Назовите ПЕРВОЕ и ВТОРУЮ. Ответ: Яблоко, лошадь. Комментарий: После Первой мировой лошадей было в достатке, а вот сельское хозяйство пришло в упадок, и достать что-нибудь вкусное было сложно. Источник: 1. http://lib.ru/INPROZ/BELL/billiard.txt 2. http://bibliogid.ru/folders/205 Автор: Дмитрий Великов Вопрос 10: В одном из кадров фильма "У матросов нет вопросов" мы видим на поверхности моря два спасательных круга. Первый из них мог принадлежать спущенному на воду 16 марта 1914 года эсминцу. Что написано на втором? Ответ: Конец. Комментарий: А на первом, соответственно, написано "Счастливый". Русские эсминцы обычно назывались прилагательными. Автор: Андрей Штефан, Елизавета Штефан Вопрос 11: В своих путевых заметках Гёте упоминает торговцев, продающих белые гипсовые шарики, похожие на драже. В каком городе сделана эта запись? Ответ: Венеция. Комментарий: Описывая венецианский карнавал, Гете замечает, что раньше в качестве конфетти действительно использовались конфеты, но теперь перешли на искусственные заменители. До бумажных конфетти в его время, очевидно, еще не дошли. Источник: И.В. Гёте. Путешествие в Италию. / Пер. Н.А. Холодковский. - М.: ОГИ, 2012 (в печати). Автор: Дмитрий Великов Вопрос 12: Автор этого вопроса иногда делает вот такой жест [залу демонстрируется сжатая в кулак рука с поднятым вверх большим пальцем]. Ответьте: чему соответствовала сначала тяжелая атлетика, а затем - теннис? Ответ: 42. Зачет: По числу без неверных уточнений; например, всякие ответы со смыслом "номер 42 в "Спортлото"". Источник: http://oldcoffer.narod.ru/1.htm Автор: Дмитрий Вайнман Вопрос 13: [Чтецу: текст раздаточного материала не зачитывать, дать командам около 15 секунд на изучение.] <раздатка> "Главный вдохновитель [ПРОПУСК], французский дипломат, в подтверждение своей буржуазной сущности имел всего-навсего один довольно помятый цилиндр, которым он и жонглировал, прикрывая брюки из чертовой кожи и холщовую рубаху".   Перед вами цитата из книги [ПРОПУСК], написанной в соавторстве. Назовите эту книгу. Назовите эту книгу. Ответ: "Республика ШКИД". Комментарий: Нами пропущены слова "бЕлых" и "БелЫх". Источник: http://lib.ru/RUSSLIT/PANTELEEW/respublikashkid.txt Автор: Дмитрий Великов Вопрос 14: В какой статье Википедии, озаглавленной словом, имеющим греческое происхождение, есть цитата из написанного в XII веке текста о слышащих, но не понимающих правильно слово "воспоминание"? Ответ: Евхаристия. Комментарий: [Зачитать.] Цитата гласит: "Слышащим спасителя о преданном Им священнодействии Божественных Таин, говорящего: "сие творите в Мое воспоминание", но не понимающим правильно слова "воспоминание" <...> анафема трижды". Это из установления вселенского собора по поводу причастия. Что до текста вопроса, то в нем на одно предложение приходится пять причастий. Цитата, которую мы здесь приводим отнюдь не полностью, в этом отношении куда как более примечательна: в ней причастий около дюжины. [Зачитывать не надо.] Цитата полностью звучит так: "Слышащим Спасителя о преданном Им священнодействии Божественных Таин, говорящего: "сие творите в Мое воспоминание", но не понимающим правильно слова "воспоминание" и дерзающим говорить, что оно (т.е. воспоминание) обновляет мечтательно и образно жертву Его Тела и Крови, принесенную на честном Кресте Спасителем нашим в общее избавление и очищение, и что оно обновляет и ежедневную жертву, приносимую священнодействующими Божественные Тайны, как предал Спаситель наш и Владыка всех, и поэтому вводящим, что это иная жертва, чем совершенная изначала Спасителем и возносимая к той мечтательно и образно, как уничижающим неизменность жертвы и таинство страшного и Божественного священнодействия, которым мы принимаем обручение будущей жизни, как это изъясняет Божественный отец наш Иоанн Златоуст во многих толкованиях посланий великого Павла, анафема трижды". Источник: http://ru.wikipedia.org/wiki/Евхаристия Автор: Дмитрий Великов Вопрос 15: Иностранные гости в одной из пьес Платонова говорят, что у них построена государственная тишина, и желают купить в СССР ЕЕ. В одной из статей Википедии про НЕЕ упоминается рубка. Назовите ЕЕ. Ответ: Надстройка. Комментарий: Так иностранцы описывают свой базис. Продали, кстати, им вместо этого лишь директивы. Источник: 1. А. Платонов. Шарманка / А. Платонов. Взыскание погибших: Повести. Рассказы. Пьеса. Статьи. - М.: Школа-пресс, 1995. - С. 546. 2. http://ru.wikipedia.org/wiki/Надстройка_(судостроение) Автор: Дмитрий Великов Тур: 16 тур. Команда Алексея Штыха Дата: 16-Jun-2012 Редактор: Алексей Трефилов (Калуга) Инфо: Редактор благодарит за тестирование и ценные замечания: Валерия Болгана, Дмитрия Борока, Галину Воловник, Артема Колесова, Александра Кудрявцева, Константина Леонченко, Максима Мерзлякова, Бориса Моносова, Константина Науменко, Александра Рождествина, Илью Севастьянова, Павла Солахяна, Григория Смыслова, Юлию Сычеву, Александра Толесникова, Игоря Тюнькина и команду "Минус один" (Киев). Вопрос 1: [Нулевой вопрос] (pic: 20110751.jpg) Этот фрагмент мы взяли с герба одного района. С какого? Ответ: Люберецкого. Комментарий: Гантель на гербе Люберецкого района символизирует спортивность местных жителей (посещающих качалку). Кстати, оформление герба самих Люберец посвящено вовсе не качкам, а Юрию Гагарину. Источник: http://geraldika.ru/symbols/954 Автор: Михаил Малкин (Люберцы, Московская область) Вопрос 2: [Нулевой вопрос] Говоря о преемственности в российской поп-музыке, Алексей Штых сравнил розовые розы с НИМ. Назовите ЕГО. Ответ: Бордовое бордо. Комментарий: Примером тавтологии служит не только строчка из ВИА "Веселые ребята", но и строчка песни Ёлки "Прованс". Источник: 1. http://www.sovetskaya-estrada.ru/load/veselye_rebjata/quot_rozovye_rozy_quot/7-1-0-170 2. http://www.sentido.ru/songs.php?id_song=1297 Автор: Алексей Штых (Москва) Вопрос 3: Известно, что Михаил Булгаков очень ценил Гоголя и равнялся на него. Это же сказалось на судьбе романа "Мастер и Маргарита". Говоря об этом, Мариэтта Чудакова замечает, что Булгаков [ПРОПУСК] Гоголя. Заполните пропуск устойчивым выражением из трех слов. Ответ: Танцевал от печки. Зачет: Плясал от печки. Комментарий: А шинель тут ни при чем. :-) Чудакова рассказывает, что Булгаков сжигал и восстанавливал части романа. Источник: http://gordon0030.narod.ru/archive/11427/index.html Автор: Даниил Марченко (Калуга) Вопрос 4: По словам Дмитрия Чернышёва, в НЕЙ есть стиль, метр и текст. Назовите ЕЕ тремя словами. Ответ: Станция метро "Текстильщики". Источник: http://mi3ch.livejournal.com/1730618.html Автор: Сергей Чериканов (Калуга) Вопрос 5: Татьяна Бочкарева упоминает ИХ, сравнивая нынешний кризис ценных бумаг с кризисом фермерских участков в Америке XIX века. Герой пьесы говорит о НИХ, когда действие происходит на болоте. Назовите ИХ двумя словами. Ответ: Пузыри земли. Комментарий: Герой "Макбета" имеет ввиду болотный газ, а Бочкарева намекает на "мыльные пузыри". Источник: 1. http://www.vedomosti.ru/newspaper/article/257342/puzyri_zemli 2. http://lib.ru/SHAKESPEARE/shks_mcbeth4.txt Автор: Александр Марков (Смоленск - Москва) Вопрос 6: Майрик и Санчес расклеивали в университетских кампусах листовки с портретами, где содержалась просьба оказать содействие полиции в поисках. Вскоре миллионы людей могли наблюдать эти же лица... Ответьте максимально точно: где именно? Ответ: В фильме "Ведьма из Блэр[: курсовая с того света]". Комментарий: Чтобы убедить, что история, рассказанная в фильме, произошла в действительности. Источник: Гладильщиков Ю. Справочник грез. Путеводитель по новому кино. - М.: КоЛибри, 2008. - С. 128. Автор: Борис Моносов (Санкт-Петербург) Вопрос 7: <раздатка> [Пропуск] never discovered gold, but he managed to discover a [пропуск] new element. Напишите современное название элемента, об открытии которого рассказывается в процитированной статье. Ответ: Фосфор. Комментарий: В первом случае пропущена фамилия немецкого алхимика Хеннинга Бранда, который открыл фосфор в 1669 году. Во втором случае пропущена часть английского выражения "brand new" [брэнд нью], которое означает "совершенно новый". Пропуски на письме отличаются только капитализацией. :-) Источник: 1. Статья автора вопроса. 2. http://en.wikipedia.org/wiki/Chemistry:_A_Volatile_History#Hennig_Brand_and_the_Icy_Noctiluca Автор: Игорь Тюнькин (Москва) Вопрос 8: Один русский писатель иронизировал, что человек, которого накормили ЭТИМ БЛЮДОМ, чувствует себя нелепо: словно сам он стоит босой, а в желудке у него - спазм. Назовите ЭТО БЛЮДО двумя словами. Ответ: Сапоги всмятку. Комментарий: Правда, этот писатель не Чехов (у которого есть рассказ "Сапоги всмятку"), а Чернышевский. Идиома "сапоги всмятку" означает "вздор, чепуха, бессмысленность". Источник: 1. http://www.gramota.ru/spravka/hardwords/25_361, 2. http://phraseology.academic.ru/11099/ Автор: Алексей Трефилов (Калуга) Вопрос 9: В романе Тома Шарпа немало литературных аллюзий. Героиня, считающая, что ее возлюбленный предпочитает женщин старше себя, замечает, что НАБОКОВ - ее любимый ПИСАТЕЛЬ. Какие слова мы заменили на "НАБОКОВ" и "ПИСАТЕЛЬ"? Ответ: Брамс, композитор. Комментарий: В отличие от героини Франсуазы Саган, героиня Шарпа Брамса любит. Источник: http://lib.ru/INPROZ/SHARP/tom_sharp_dal_nij_umysel.txt Автор: Александр Марков (Смоленск - Москва) Вопрос 10: Евгений Лукин рассказывает, что газетная колонка, в которой он публиковал критические стихотворения, называлась "ИКС всему". По одной версии, Маяковский начал писать "Стихи о советском паспорте" в ИКСАХ. Какое слово мы заменили на "ИКС"? Ответ: Столбец. Комментарий: По одной версии, Маяковский начал писать стихотворение, в котором рассказывается о пересечении границы, в городе Столбцы, находившемся на старой Советско-Польской границе. А во время пересечения границы поэт был в одном купе с иностранцами. Источник: 1. http://www.litmir.net/br/?b=33953&p=10 2. http://ru.wikipedia.org/wiki/Столбцы 3. http://www.newsvm.com/news/26/46593/ Автор: Алексей Трефилов (Калуга) Вопрос 11: Когда гости из США спросили архитектора Евгения Асса, как можно жить в квартире с окнами на Садовое кольцо, он посоветовал побывать ТАМ. Одна из главных проблем ТАМ - образование плесени. Где ТАМ? Ответ: В Доме над водопадом [Фрэнка Райта]. Комментарий: Архитектор спросил, были ли американцы в знаменитой постройке Райта - находящемся в Пенсильвании Доме над водопадом, в котором постоянно слышен шум падающей воды. Источник: 1. http://www.afisha.ru/article/progulka-ass/page2/ 2. http://en.wikipedia.org/wiki/Fallingwater Автор: Михаил Малкин (Люберцы, Московская область) Вопрос 12: <раздатка> Но как воззову я к Богу моему, к Богу и Господу моему? Когда я воззову к нему, я призову Его в самое себя. Где же есть во мне место, куда пришел бы господь Мой? ОНО упоминается в произведении первой половины XIX века. Розданный отрывок - ОНО. Некое слово в последнем предложении отрывка - тем более ОНО. Назовите ЕГО пятью словами. Ответ: Одно место из Блаженного Августина. Комментарий: Упоминается в "Трех мушкетерах" как вымышленная причина дуэли д'Артаньяна и Арамиса. Источник: 1. Аврелий Августин. Исповедь. - СПб.: Азбука, 1999. - С. 6. 2. http://ru.wikipedia.org/wiki/Три_мушкетёра Автор: Артем Ковалев (Калуга) Вопрос 13: Любопытно, что применительно к известному советскому произведению в широком смысле можно употребить термин "спин-офф". В этом произведении ИКС винит ИКС, что в самом начале операции оказался вдалеке от своих сослуживцев. Какое слово мы заменили на "ИКС"? Ответ: Вихрь. Комментарий: Английское слово spin означает "вращение, кружение и т.п.". Спин-оффом также называют произведение, вселенная которого или персонажи были ранее использованы в другом произведении. Роман "Майор Вихрь" является частью цикла о Штирлице. Но центральным персонажем книги является не сам Штирлиц, а группа диверсантов (которую сбрасывают на парашютах в тыл врага), один из бойцов группы - сын Максима Исаева, Александр. Майор Вихрь размышляет: "... а они прыгали первыми, следовательно, они приземлились в том направлении, откуда налетал ветер. Вихрь, - усмехнулся он, - налетал вихрь...". Источник: 1. http://lingvo.yandex.ru/spin/с%20английского/ 2. http://ru.wikipedia.org/wiki/Спин-офф 3. http://en.wikipedia.org/wiki/Spin-off_(media) 4. http://ru.wikipedia.org/wiki/Майор_Вихрь_(роман) 5. http://lib.ru/RUSS_DETEKTIW/SEMENOW_YU/vihre.txt Автор: Алексей Трефилов (Калуга) Вопрос 14: В одном фильме действует тайная организация, которая вознаграждает людей за хорошие поступки и наказывает за плохие. Название этого фильма совпадает с названием произведения, впервые исполненного в 1996 году. Напишите это название. Ответ: "Karma Police". Зачет: "Полиция кармы"; "Кармическая полиция". Комментарий: Песня "Radiohead". Источник: 1. http://www.kinopoisk.ru/level/1/film/312722/ 2. http://en.wikipedia.org/wiki/Karma_Police Автор: Александр Марков (Смоленск - Москва) Вопрос 15: В знаменитой пьесе семнадцатого века одним из персонажей является отец молодой дочери. В кульминационной сцене четвертого действия он, заранее спрятавшись, подслушивает беседу заглавного героя с другой героиней. Имя этого персонажа на слух можно принять за название химического элемента. Какого? Ответ: Аргон. Комментарий: Жена уговаривает Оргона спрятаться под столом, чтобы убедиться в том, какой Тартюф лицемер. Полоний подслушивает разговор Гамлета с Гертрудой в третьем акте, а не в четвертом. :-) Источник: 1. http://lib.ru/MOLIER/tartuf2.txt 2. http://lib.ru/SHAKESPEARE/hamlet5.txt Автор: Борис Моносов (Санкт-Петербург) Вопрос 16: <раздатка> Придет день, когда само изображение, вместе с тем именем, которое оно носит, будет деидентифицировано подобием, бесконечно разносящимся на всем протяжении серии. Бограч, доматес чорба, харира, таматар ка. В цитате из эссе Мишеля Фуко мы сделали несколько замен. Восстановите первоначальный текст. Притяжательный падеж использовать не нужно. Ответ: Campbell, Campbell, Campbell, Campbell. Зачет: Кэмпбелл, Кэмпбелл, Кэмпбелл, Кэмпбелл. Комментарий: Цитата заканчивается четырехкратным повторением слова "Кэмпбелл" - намек на работы Энди Уорхола, использовавшего многократно повторяющиеся изображения банки томатного супа "Campbell's". Бограч, доматес чорба, харира, таматар ка - это томатные супы. Источник: 1. М. Фуко. Это не трубка. http://www.flibusta.net/b/138351/read 2. http://ru.wikipedia.org/wiki/Уорхол,_Энди 3. http://uk.wikipedia.org/wiki/Бограч 4. http://www.babyblog.ru/community/post/national_cooking/460584 5. http://www.emirat.ru/wiki/Рестораны 6. http://www.womanline.ru/458-tamatar_ka_sup_tomatnyj Автор: Максим Мерзляков (Воронеж) Вопрос 17: Недавно автор вопроса прочитал заметку об уборщике в палеонтологическом музее. Этот уборщик всего лишь стряхивал пыль с экспонатов. Но журналист вынес в заглавие заметки идиому. Видать, ему в голову ничего лучше не пришло, как приврать ради красного словца. Напишите эту идиому из двух слов. Ответ: Перемывать кости. Комментарий: Уборщик не перемывал кости динозаврам, а только стряхивал с костей пыль. Зато автор вопроса перемыл кости журналисту. :-) А игроки теперь могут перемыть кости редактору и авторам вопросов. :-) Источник: http://www.vokrugsveta.ru/vs/article/7469/ Автор: Алексей Трефилов (Калуга)